Você está na página 1de 45

DESGLOSES

CTO Medicina
DESGLOSES MIR

ISBN: 978-84-16153-89-3
MIR
DESGLOSES
Introducción: P076 MIR 2012-2013

T01 anatomía, semiología


y fisiología del sistema Ante un paciente que presenta problemas para
nervioso la comprensión del lenguaje tanto hablado
como escrito, incapacidad para denominar ob-
P060 MIR 2013-2014 jetos y repetir palabras que se le dicen, habla
fluida incomprensible con parafasias semánti-
Juan tiene 60 años, fuma de 2 paquetes/día cas y fonémicas, se trata de una:
desde hace años y refiere desde hace 6 meses
tos persistente. Comprueba que su párpado iz- 1) Afasia global.
quierdo está más caído y que la pupila de este 2) Afasia de Wernicke.
ojo es más pequeña. Juan refiere que la parte 3) Afasia de Broca.

MIR
medial de su mano izquierda está adormecida 4) Afasia transcortical sensitiva.
y con menos fuerza. Su médico comprueba 5) Afasia transcortical motora.
la ptosis palpebral y la miosis izquierda; com-
prueba que puede cerrar con fuerza ambos Respuesta correcta: 2
párpados simétricamente y que las dos pupi-
las responden correctamente a la luz. Además
comprueba que no suda por la hemicara iz- P146 MIR 2012-2013
quierda, que siente memos el pinchazo en la
superficie interna de dicha mano y que tiene Paciente de 61 años con antecedentes de HTA
menos fuerza en la prensión de dicha mano. y Diabetes Mellitus que consulta por diplopía
Respecto a la sintomatología ocular, ¿dónde se de inicio brusco. En la exploración oftalmoló-
localiza la lesión? gica presenta ptosis en ojo derecho, limitación
de la aducción, supra e infraducción y movi-
1) Las fibras simpáticas, en algún nivel que abar- mientos pupilares conservados. El diagnóstico

Neurología caría desde el hipotálamo a la columna de


Clark intermedio-lateral de la médula dorsal.
mas probable es:

2) El nervio motor ocular común izquierdo en el 1) Aneurisma de la arteria comunicante posterior.

y neurocirugía mesencéfalo.
3) El núcleo de Edinger-Westphal, encima del
2)
3)
Parálisis postraumática.
Isquemia microvascular.
núcleo del nervio motor ocular común iz- 4) Neoplasia.
quierdo. 5) Migraña oftalmopléjica.
4) Las fibras parasimpáticos, en algún nivel que
abarcaría desde el núcleo de Edinger-Wes- Respuesta correcta: 3
tphal hasta el músculo constrictor de la pupila
izquierda.
5) El músculo tarsal exclusivamente. P200 MIR 2012-2013

Respuesta correcta: 1 Juan tiene 60 años, fuma 2 paquetes/día desde


hace años y refiere desde hace 6 meses tos per-
sistente. Comprueba que su párpado izquier-
P145 MIR 2013-2014 do está más caído y que la pupila de ese ojo es
más pequeña. Juan refiere que la parte medial
¿Cuál de los siguientes signos clínicos NO se de su mano izquierda está adormecida y con
observa en las lesiones de la neurona motora menos fuerza. Su médico comprueba la ptosis
inferior? palpebral y la miosis izquierdas; comprueba
que puede cerrar con fuerza ambos párpados
1) Parálisis. simétricamente y que las dos pupilas respon-
2) Amiotrofia. den correctamente a la luz. Además comprue-
3) Fasciculaciones. ba que no suda por la hemicara izquierda, que
4) Arreflexia. siente menos el pinchazo en la superficie inter-
5) Hipoestesia. na de dicha mano y que tiene menos fuerza en
la prensión de dicha mano. ¿A qué nivel tiene
Respuesta correcta: 5 el enfermo lesionada la mano izquierda?
DESGLOSES MIR
1) A nivel del pedúnculo cerebral ipsilateral tras 1) Neuropatía periférica sensitiva simétrica distal 4) Mononeuropatía múltiple con afectación de N.
la decusación de las pirámides bulbares, por de causa diabética. peroneo común y tibial posterior derecho.
invasión tumoral o absceso. 2) Lesión compresiva medular cervical. 5) Plexopatía lumbar derecha.
2) A algún nivel del tracto corticoespinal derecho, 3) Síndrome del túnel del carpo bilateral.
por metástasis pulmonar. 4) Enfermedad desmielinizante tipo esclerosis Respuesta correcta: 2
3) A nivel de las raíces espinales cervicales inferio- múltiple.
res al entrar en el plexo braquial izquierdo, por 5) Lesión medular central cervical.
invasión de un tumor del vértice pulmonar. P220 MIR 2010-2011
4) A nivel del nervio mediano izquierdo, por inva- Respuesta correcta: 5
sión de un tumor del vértice pulmonar. Durante una intervención neuroquirúrgica, la
5) A nivel bulbar lateral izquierdo, por un infarto estimulación eléctrica cortical directa produjo
de la arteria cerebelosa posteroinferior. P208 MIR 2011-2012 un “movimiento de prensión de ambas manos”.
¿Qué área cortical estimulada eléctricamente
Respuesta correcta: 3 Un individuo presenta “debilidad muscular” (he- produce esta respuesta?
miparesia espástica) de ambas extremidades
derechas, con hiperreflexia y signo de Babinski, 1) Área motara primaria.
P208 MIR 2012-2013 junto a una “parálisis flácida facial” de la hemi- 2) Área motora secundaria.
cara izquierda, con incapacidad para cerrar el 3) Área premotora.
¿Cuál es base funcional a nivel medular del do- ojo izquierdo o de retraer el lado izquierdo de 4) Área motora suplementaria.
lor referido? la boca, además de otras alteraciones. Por los 5) Área parietal de asociación.
datos descritos se trata de una alteración que
1) La activación exclusiva de neuronas específi- afecta, entre otros elementos, a los fascículos Respuesta correcta: 4
cas de la nocicepción. motores: córtico-espinal y córtico-nuclear, pero
2) La convergencia de información cutánea y vis- ¿a qué nivel del neuroeje localizaría la lesión?
ceral en neuronas de gama dinámica ancha. P222 MIR 2009-2010
3) El bloqueo de substancia P desde las aferen- 1) A nivel del área 4 de Brodmann de la corteza
cias primarias. cerebral del lado derecho. Una lesión periférica del III par craneal, nervio
4) La interrupción de las fibras del tracto espino- 2) En la cápsula interna, brazo posterior del lado oculomotor (motor ocular común) suele llevar
talámico lateral. derecho. consigo alteración de sus fibras motoras visce-
5) La separación de la información medular de 3) En el pedúnculo cerebral izquierdo. rales (parasimpáticas) al que están asociadas.
acuerdo al principio de divergencia. 4) En la porción medial de la protuberancia cau- Si estas fibras se lesionan, el paciente manifies-
dal del lado izquierdo. ta alteraciones que afectan a los reflejos ocu-
Respuesta correcta: 2 5) En el bulbo raquídeo antes de la decusación lares. De las respuestas dadas a continuación
del fascículo córtico-espinal derecho. sólo una es verdadera. ¿Cuál es?

P076 MIR 2011-2012 Respuesta correcta: 4 1) Presenta pupila contraída (en miosis) y conser-
vado el reflejo de acomodación.
Un paciente de 32 años, diabético en trata- 2) Su pupila está contraída (en miosis) y abolido
miento con insulina y un buen control de sus P068 MIR 2010-2011 el reflejo de acomodación.
cifras de glucemia, acude a su consulta por 3) Se encuentra dilatada su pupila (en midriasis)
presentar hormigueo en ambas manos, con Un paciente de 56 años sin antecedentes per- fija y conservado el reflejo de acomodación.
sensación de acorchamiento e insensibilidad sonales ni familiares de interés es estudiado 4) Presenta pupila dilatada (en midriasis) fija y
térmica de instauración progresiva en el curso por presentar desde hace 6 meses dificultad abolido el reflejo de acomodación.
de 2 semanas. No refiere trastornos visuales, para andar con la pierna derecha. No refiere 5) Su pupila no está fija (a veces dilatada y a ve-
déficit de fuerza, torpeza motora, ni otros sín- otros síntomas. La exploración muestra una ces contraída) y el reflejo de la acomodación
tomas. En la exploración encuentra una anes- debilidad 4/5 para la flexión dorsal y eversión es normal.
tesia para el dolor y la temperatura en ambas del pie y 4/5 para flexión e inversión del pie,
manos y porción distal de los antebrazos; la estando los reflejos musculares aumentados y Respuesta correcta: 4
sensibilidad posicional y la vibratoria están con Babinski presente, siendo el resto normal.
conservadas. No presenta atrofia muscular ni El diagnóstico sindrómico sería:
déficit de fuerza. Los reflejos musculares son P224 MIR 2009-2010
normales y simétricos. No se observa disme- 1) Mononeuropatía del N. peroneo común derecho.
tría, disdiadococinesia ni temblor intencional. 2) Afectación focal de primera motoneurona y El “brazo posterior de la cápsula interna” con-
El resto de la exploración neurológica es lige- probable de segunda. tiene fibras de proyección de naturaleza moto-
ramente normal. Indique cuál es el diagnóstico 3) Afectación hemimedular derecho (Brown- ra y fibras de conexión o radiaciones talámicas,
más probable en este caso: Sequard). de naturaleza sensitiva. ¿Qué fibras de las cita-

NEUROLOGÍA 260
Y NEUROCIRUGÍA
Neurología y neurocirugía
das a continuación se localizan en este “brazo 1) Mesencéfalo lateral derecho. 3) Neuropatía periférica.
posterior”? 2) Mesencéfalo medial izquierdo. 4) Atrofia por desuso.
3) Protuberancia lateral izquierda. 5) Tabes dorsal.
1) Fibras córtico putaminales. 4) Protuberancia medial derecha.
2) Fibras de las radiaciones talámicas anteriores. 5) Bulbo medial derecho. Respuesta correcta: 4
3) Fibras fronto pónticas.
4) Fibras córtico espinales. Respuesta correcta: 3
5) Fibras córtico nucleares.

Respuesta correcta: 4 P054 MIR 2004-2005 T02 Coma.


Muerte encefálica
Un paciente presenta de forma brusca un trans-
P052 MIR 2007-2008 torno del lenguaje caracterizado por lenguaje P221 MIR 2013-2014
espontáneo escaso, casi mutismo, dificultad
En relación con la patología del nervio óptico y para evocar palabras, muy leve transtorno de ¿Cuál de estas afirmaciones es FALSA respecto
los nervios oculomotores, es cierto: la comprensión, repitiendo correctamente. Su al estupor?
primera sospecha diagnóstica será:
1) Un infarto de la corteza occipital produce una 1) Es una alteración de la conciencia.
hemianopsia homónima ipsilateral. 1) Cuadro confusional agudo. 2) Puede darse en la melancolía.
2) La duración más habitual de la amaurosis fu- 2) Accidente isquémico frontal profundo izquier- 3) Supone un estado reversible por estímulos
gaz es de entre 45 y 60 minutos. do. moderados.
3) En la neuritis óptica las pupilas son isocóricas 3) Hemorragia subaracnoidea. 4) Se utiliza el término en estados de mutismo y
y puede encontrarse un defecto pupilar afe- 4) Infarto silviano derecho. reducción de la actividad motora.
rente. 5) Lesión del cuerpo calloso. 5) En neurología es un estado que precede al
4) La pérdida de agudeza visual en el edema de coma.
papila es muy grave. Respuesta correcta: 2
5) En las lesiones compresivas del III par (motor Respuesta correcta: 3
ocular común) hay característicamente un res-
peto de la función pupilar. P057 MIR 2004-2005
P067 MIR 2011-2012
Respuesta correcta: 3 Se observa que un hombre de 80 años presen-
ta una marcha lenta de base ancha al andar Existen varios términos semiológicos que tie-
desde la sala de espera a la de reconocimiento nen en cuenta el patrón respiratorio de un
P053 MIR 2005-2006 para una evaluación rutinaria. Niega cualquier paciente. Si se observa en un paciente, tras un
problema especial con la marcha, aunque re- periodo de apnea, que se producen respira-
En la enfermedad oclusiva del sistema arterial fiere caídas ocasionales no asociadas con una ciones que van aumentando progresivamente
vértebro-basilar, ¿cuál de los siguientes signos lesión. No presenta diabetes y se somete a se- de amplitud y frecuencia, y luego disminuyen
NO es característico? guimiento sólo debido a una leve hipertensión progresivamente hasta otro episodio de ap-
controlada mediante dieta. En raras ocasiones nea. ¿A qué patrón corresponde?
1) Meningismo. practica ejercicio. En el examen, presenta una
2) Debilidad de miembros. leve debilidad de los músculos cuádriceps 1) Respiración de Cheyne-Stokes.
3) Ataxia de la marcha. (4+/5 manualmente); un ángulo de movimien- 2) Taquipnea.
4) Parálisis oculomotora. to normal de las articulaciones; 1 + reflejos de 3) Platipnea.
5) Disfunción orofaríngea. los tobillos; una propiocepción un poco dis- 4) Respiración de Kussmaul.
minuida, pero presente; ninguna disminución 5) Bradipnea.
Respuesta correcta: 1 obvia de la sensibilidad; y una pequeña vacila-
ción al levantarse de la silla. La prueba de Rom- Respuesta correcta: 1
berg es normal. Anda sin ningún dispositivo
P054 MIR 2005-2006 de ayuda, pero su manera de andar es lenta y
cautelosa, con pasos reducidos y un pequeño P053 MIR 2004-2005
Un paciente de 62 años presenta de forma ensanchamiento de la base del soporte. ¿Cuál
brusca una hemihipoestesia termoalgésica del es la causa más probable de la dificultad de la Hombre de 57 años que ingresa en el Servi-
hemicuerpo derecho, así como hipoestesia de marcha de este paciente? ciode Urgencias de nuestro hospital, tras ser
la hemicara izquierda, hemiataxia izquierda y encontrado en la calle sin respuesta a estímu-
debilidad de los músculos de la masticación. 1) Enfermedad de Parkinson. los. En la exploración física a su llegada desta-
¿Dónde localizaremos la lesión? 2) Osteoartritis. ca coma con escala de Glasgow de 3 puntos,

261
DESGLOSES MIR
pupilas puntiformes, reflejo corneal abolido, P152 MIR 2013-2014 P079 MIR 2011-2012
respiración de Kussmaul y sudoración intensa.
El cuadro no se modifica tras administración En el tratamiento de los pacientes afectos de Un varón de 78 años consulta por un cuadro
de 0,4 mg de naloxona intravenosa. Señale el enfermedad de Alzheimer, ¿cuál de las siguien- de deterioro cognitivo progresivo de un año
diagnóstico más probable, entre los siguien- tes opciones terapéuticas considera más ade- de evolución con fallos de memoria y de orien-
tes: cuada? tación. Su familia refiere alucinaciones visuales
recurrentes, caídas ocasionales y un enlenteci-
1) Infarto en el territorio de la arteria cerebral me- 1) Es de elección el tratamiento inicial de cual- miento motor llamativo. ¿Cuál es el diagnósti-
dia derecha. quier tipo de incontinencia urinaria con fár- co más probable?
2) Intoxicación por cocaína. macos como la oxibutinina, por sus efectos
3) Hemorragia pontina. anticolinérgicos. 1) Demencia vascular multi-infarto.
4) Hemorragia talámica izquierda. 2) El tratamiento con inhibidores de la acetilco- 2) Demencia tipo Alzheimer.
5) Intoxicación por opiáceos. linesterasa. 3) Demencia con cuerpos de Lewy difusos.
3) Para el tratamiento de la depresión asociada prio- 4) Demencia fronto-temporal.
Respuesta correcta: 3 rizar la utilización de antidepresivos tricíclicos. 5) Hidrocefalia nomotensiva.
4) Los neurolépticos típicos a dosis elevadas.
5) La terapia de reemplazo con estrógenos. Respuesta correcta: 3

T03 Demencias Respuesta correcta: 2


P067 MIR 2010-2011

P143 MIR 2013-2014 P073 MIR 2012-2013 El gen de la apolipoproteína épsilon, en el cro-
mosoma 19, posee tres alelos, E2, E3, E4. Se
Hombre de 76 años de edad que consulta por Un hombre de 77 años, con antecedentes de conoce que interviene en el transporte del co-
deterioro cognitivo, lentitud y torpeza de mo- Diabetes Mellitus tipo 2, fue diagnosticado lesterol, su forma E4 se considera un factor de
vimientos, de ocho meses de evolución. La hace 2 años de deterioro cognitivo leve. En riesgo para:
familia refiere que el paciente presentaba alu- aquel momento se practicó un Minimental
cinaciones visuales, por lo que su médico de (Folstein) 28/30, Test de Yesavage abreviado 1) Enfermedad de Alzheimer.
Atención Primaria pautó dosis bajas de rispe- 14/15, una analítica con TSH y factores de 2) Enfermedad de Parkinson.
ridona, con un importante empeoramiento del maduración normales junto con serologías 3) Corea de Huntington.
estado motor. A la vista de estos datos, ¿cuál es de LUES y VIH negativas y una resonancia 4) Enfermedad de CADASIL (arteriopatía cerebral
el diagnóstico más probable? magnética (RM) que mostraba atrofia corti- autosómica dominante con infartos cerebrales
cal difusa predominante en zona posterior. y leucoencefalopatía).
1) Enfermedad de Alzheimer. El paciente acude acompañado de su esposa, 5) Ataxia-telangiectasia.
2) Demencia por cuerpos de Lewy. que refiere evolución progresiva y lenta del
3) Demencia frontotemporal. deterioro de la memoria (p.e. en ocasiones se Respuesta correcta: 1
4) Demencia vascular. ha confundido con las dosis de insulina). No
5) Enfermedad de Creutzfeldt-Jakob esporádi- existen alucinaciones ni trastorno conductual.
ca. El Minimental actual es de 24/30 y el Test de P069 MIR 2010-2011
Yesavage 14/15, no existe focalidad neuroló-
Respuesta correcta: 2 gica, temblor ni trastornos del tono o de la En relación con la demencia con cuerpos de
marcha en la exploración física. Hace 2 meses Lewy indique la opción falsa:
acudió a Urgencias tras un traumatismo crá-
P148 MIR 2013-2014 neoencefálico leve por caída accidental (la 1) Son frecuentes las fluctuaciones cognitivas y
única en 2 años) y se realizó una tomografía atencionales.
¿Cuál de estos procesos es una manifestación que no aportaba nueva información respec- 2) El parkinsonismo se incluye entre los criterios
de la enfermedad de pequeño vaso (arterias to a la RM previa. ¿Cuál es el diagnóstico más diagnósticos.
perforantes cerebrales)? probable en este paciente? 3) Los síntomas psicóticos son frecuentes, predo-
minando las alucinaciones visuales.
1) Hemorragia lobar. 1) Demencia frontotemporal. 4) Los neurolépticos deben utilizarse en la mayo-
2) Desmielinización isquémica (leucoaraiosis). 2) Enfermedad de Alzheimer. ría de pacientes para controlar los trastornos
3) Infarto talámico. 3) Demencia vascular. conductuales.
4) Neuropatía óptica isquémica. 4) Enfermedad por cuerpos de Lewy. 5) La respuesta a la levodopa suele ser escasa y
5) Arteritis necrosante. 5) Enfermedad de Creutzfeldt-Jakob. transitoria.

Respuesta correcta: ANU Respuesta correcta: 2 Respuesta correcta: 4

NEUROLOGÍA 262
Y NEUROCIRUGÍA
Neurología y neurocirugía
P147 MIR 2010-2011 vía en una fase leve, implican la memoria, las
habilidades matemáticas, la orientación y la T04 Enfermedades
vasculares cerebrales
Un hombre de 67 años acude a la consulta capacidad de aprender nuevas habilidades.
presentando un deterioro cognitivo progre- Aunque se han observado fluctuaciones en
sivo y de rápida evolución (meses) que afecta la capacidad cognitiva día a día o semana P099 MIR 2013-2014
preferentemente a las funciones corticales a semana, es aparente un curso de declive
prefrontales (fluencia verbal, funciones ejecu- definido. El paciente no ha estado tomando Acude a nuestra consulta una mujer de 70
tivas) y parietales (función visual-constructiva). ningún medicamento. El examen físico revela años preocupada por su riesgo de sufrir un
También destaca un déficit en la capacidad un temblor en reposo de la rigidez en rueda accidente cerebrovascular, ya que su madre
atencional, preservación de la capacidad mné- dentada. La marcha del paciente se caracte- falleció por esta causa hace un año. Tiene his-
sica, síntomas psicóticos que incluyen alucina- riza por pasos cortos que los arrastra y una toria de hipertensión arterial y diabetes melli-
ciones visuales y olfativas, e ideas delirantes disminución del balanceo de los brazos. ¿Qué tus tipo 2 por lo que está en tratamiento con
secundarias, síntomas extrapiramidales (par- síndrome clínico es más compatible con los glipizida, aspirina, enalapril y atorvastatina.
kinsonismo acinético) y neurovegetativos, al- síntomas de este hombre? Fuma 20 cigarrillos al día y no realiza ejercicio
teraciones en la marcha y notable fluctuación de forma regular. A la exploración se detecta
de la gravedad de la clínica de un día para otro. 1) Enfermedad de Parkinson. una presión arterial de 150/80 mmHg. En la
El cuadro clínico sugiere: 2) Demencia de Creutzfeldt-Jacob. analítica destaca una hemoglobina A1c de 8%
3) Demencia con cuerpos de Lewy. y un LDL colesterol de 110 mg/dl. ¿Cuál de los
1) Una demencia tipo Alzheimer. 4) Demencia váscular. siguientes se asocia con una mayor reducción
2) Una demencia vascular. 5) Demencia del lóbulo frontal. del riesgo de ACV?
3) Una demencia-SIDA.
4) Una demencia por cuerpos de Lewy. Respuesta correcta: 3 1) Conseguir unos niveles óptimos de hemo-
5) Una demencia de Pick. globina A1c.
2) Conseguir un óptimo control de la presión ar-
Respuesta correcta: 4 P233 MIR 2005-2006 terial.
3) Añadir al tratamiento un antioxidante.
En algunas enfermedades neurodegenerativas 4) Abandonar el tabaco.
P228 MIR 2008-2009 se produce acúmulo intracelular de proteínas 5) Conseguir unos niveles óptimos de LDL.
anormales. Una de estas enfermedades, la en-
En la enfermedad de Alzheimer se produce fermedad de Alzheimer, se caracteriza por el Respuesta correcta: 2
una degeneración de la corteza cerebral. Las depósito intracelular de:
lesiones que se observan en la corteza cere-
bral, además de las placas seniles, son: 1) Sinucleina. P151 MIR 2013-2014
2) Tau hiperfosforilada.
1) Degeneración neurofibrilar. 3) Huntingtina. Un hombre de 80 años es ingresado por un
2) Cuerpos de Lewy. 4) Ataxina. cuadro brusco de afasia y hemiparesia dere-
3) Espongiosis. 5) Proteína priónica. cha. Como antecedentes destaca hipertensión,
4) Infiltración inflamatoria. bien controlada con dieta y deterioro cognitivo
5) Cuerpos de Lafora. Respuesta correcta: 2 en el último año en estudio por su neurólogo.
La TC craneal de urgencias demuestra un he-
Respuesta correcta: 1 matoma lobar frontal izquierdo sin captación
P250 MIR 2003-2004 de contraste. ¿Cuál es la causa más probable
del hematoma?
P059 MIR 2005-2006 Todas las siguientes pruebas complementarias
son recomendadas en la rutina diagnóstica de 1) Malformación arteriovenosa enmascarada por
Un hombre de 77 años es traído a la consul- la demencia, menos una. ¿Cuál es ésta? el hematoma agudo.
ta por su esposa para evaluación. Ella refiere 2) Hipertensión arterial crónica.
que durante los últimos seis meses su mari- 1) Electrolitos séricos. 3) Vasculitis aislada del sistema nervioso.
do ha experimentado fuertes alucinaciones 2) Serología luética. 4) Tumor cerebral.
visuales y auditivas e ideas delirantes para- 3) TAC cerebral. 5) Angiopatía cerebral amiloidea (angiopatía con-
noides. Asimismo, durante el último año los 4) Genotipo ApoE. gofílica).
déficits cognitios progresivos se han vuelto 5) Función tiroidea.
cada vez más evidentes para ella y para otros Respuesta correcta: 5
miembros de la familia. Estos déficits, toda- Respuesta correcta: 4

263
DESGLOSES MIR
P077 MIR 2012-2013 P077 MIR 2011-2012 4) Tratamiento con anticoagulación con hepari-
na sódica.
Indique, entre las siguientes, cuál es la manifes- Indique qué arteria es la afectada en un pa- 5) Iniciaría tratamiento con fármacos antiepilépti-
tación clínica MENOS frecuente de los infartos ciente diestro que, por un accidente cerebro- cos para la profilaxis de crisis epilépticas.
lacunares: vascular, sufre trastornos motores, sensitivos y
afasia: Respuesta correcta: 2
1) Hemiplejia.
2) Disartria. 1) Cerebral anterior derecha.
3) Ataxia. 2) Cerebral media derecha. P070 MIR 2010-2011
4) Afasia. 3) Cerebral media izquierda.
5) Déficit sensitivo. 4) Cerebral anterior izquierda. El tratamiento fibrinonilítico con R-TPA por vía
5) Cerebral posterior. i.v. está indicado en los pacientes con ictus is-
Respuesta correcta: 4 quémico agudo. ¿Cuál es el tiempo de inicio
Respuesta correcta: 3 de tratamiento que ha demostrado ser eficaz?

P078 MIR 2012-2013 1) Durante la primera semana.


P015 MIR 2010-2011 2) Durante las primeras 24 horas.
¿En cuál de las siguientes situaciones estaría in- 3) No existe un tiempo límite para iniciar el tra-
dicada la fibrinolisis endovenosa para tratar un Pregunta vinculada a la imagen n.º 8 tamiento.
infarto cerebral? 4) Durante las primeras 12 horas.
Un hombre de 55 años acude a Urgencias por 5) Durante las primeras 3 horas.
1) Hombre de 73 años de edad con antecedentes presentar hacía 45 minutos y de forma brusca
de hipertensión arterial, clínica de hemiparesia una hemiparesia izquierda. Se solicita una TC Respuesta correcta: 5
derecha y afasia de 2,5 horas de evolución y TC craneal que es la que ve en la Figura 8. ¿Cuál de
craneal normal. las siguientes consideraciones en el diagnóstico
2) Hombre de 91 años de edad con anteceden- le parece la más correcta? P051 MIR 2009-2010
tes de demencia tipo Alzheimer y dependien-
te para la mayoría de las actividades de la vida 1) La causa más probable es la hipertensión ar- La indicación más aceptada de tratamiento qui-
diaria, con clínica de hemiparesia derecha y terial. rúrgico o endovascular de las estenosis de la ar-
afasia de 1 hora y 30 minutos de evolución. En 2) Se trata de una hemorragia lobar secundaria a teria de carótida interna extracraneal a nivel de
el TC craneal se evidencia discreta atrofia cere- angiopatía congófila por amiloidosis. la bifurcación asintomática es cuando la arteria
bral global. 3) El paciente presenta un absceso cerebral posi- presenta:
3) Hombre de 37 años de edad, sin ningún an- blemente secundario a toxoplasmosis.
tecedente de interés, con clínica de cefalea, 4) El paciente presenta un ictus isquémico en el 1) Estenosis del 50% de la luz de la arteria.
hemiparesia y hemihipoestesia izquierda de territorio profundo de la arteria cerebral media 2) Estenosis del 30% de la luz.
30 minutos de evolución en el que no se derecha. 3) Estenosis del 90% de la luz.
puede realizar TC craneal por motivos técni- 5) El paciente presenta una trombosis del seno 4) Estenosis del 70% de la luz.
cos. longitudinal superior. 5) Estenosis del 10% de la luz.
4) Mujer de 53 años con antecedentes de dia-
betes mellitus tipo 2 y hemiparesia e hemihi- Respuesta correcta: 1 Respuesta correcta: 4
poestesia derecha, acompañada de disartria
de instauración brusca y que, a partir de 1 hora
y 50 minutos de inicio de los síntomas, inicia P016 MIR 2010-2011 P061 MIR 2009-2010
una mejoría espontánea del déficit neurológi-
co hasta su total resolución. Pregunta vinculada a la imagen n.º 8 En relación con la patología cerebrovascular, se-
5) Hombre de 73 años de edad con anteceden- ñale la respuesta FALSA:
tes de hipertensión arterial y con clínica de En relación con el tratamiento que indicaría para
hemiparesia derecha y afasia de 2,5 horas de este paciente, ¿cuál le parece el más apropiado? 1) La incidencia de ictus es de 200/100.000 habi-
evolución y TC craneal con lesión ocupante tantes/año.
de espacio que capta contraste en anillo, con 1) Cirugía de evacuación. 2) La causa más frecuente no traumática de he-
edema perilesional y herniación subfalcial in- 2) Control de la tensión arterial, de la glucemia y morragia subaracnoidea es la rotura de un
cipiente. de la fiebre. aneurisma.
3) Instauraría tratamiento fibrinolítico puesto que 3) Los infartos lacunares son más frecuentes en
Respuesta correcta: 1 la clínica tiene menos de tres horas de evolución. pacientes hipertensos y en diabéticos.

NEUROLOGÍA 264
Y NEUROCIRUGÍA
Neurología y neurocirugía
4) La fibrilación auricular es la causa más frecuen- P222 MIR 2008-2009 prevención de nuevos episodios su tratamien-
te de ictus cardioembólico. to inicial sería:
5) La incidencia de ictus de territorio carotídeo y ¿Dónde se origina la arteria coroidea anterior?
vértebrobasilar es prácticamente superponi- 1) Acenocumarol, ajustándolo a la dosis que con-
ble. 1) Arteria carótida común. siga un INR de 2,5.
2) Arteria carótida interna. 2) Ácido acetilsalicílico: 300 mg al día.
Respuesta correcta: 5 3) Arteria cerebral media. 3) Clopidogrel: 75 mg al día.
4) Arteria cerebral anterior. 4) Angioplastia transluminal percutánea carotí-
5) Arteria comunicante posterior. dea izquierda.
P063 MIR 2009-2010 5) Endarterectomía carotídea izquierda.
Respuesta correcta: 2
Hombre de 73 años, fumador de 40 cigarrillos al Respuesta correcta: 1
día, diabético e hipertenso mal controlado, sin
antecedentes de ictus. Acude a Urgencias por un P053 MIR 2007-2008
cuadro de 6 horas de evolución que aúna los si- P055 MIR 2006-2007
guientes síntomas y signos: vértigo, visión doble, ¿En qué situación estaría contraindicada la fi-
ptosis y miosis del ojo izquierdo, disartria, disfagia, brinólisis endovenosa para tratar un infarto Mujer de 42 años sin antecedentes de interés
ronquera, entumecimiento del lado izquierdo de cerebral? que acude a Urgencias refiriendo haber sufrido
la cara y de los miembros derechos. La TC craneal cefalea intensa de aparición súbita que persiste
muestra una marcada leucoaraiosis. La analítica 1) Edad superior a 70 años. después de seis horas y no responde a los anal-
es anodina. La radiografía de tórax no aporta da- 2) Evolución de la clínica de más de dos horas. gésicos habituales. En la exploración está afebril,
tos de interés y el ECG muestra un ritmo sinusal 3) Historia de tratamiento hipotensor. consciente y orientada, sin signos de focalidad
y signos de hipertrofia ventricular izquierda. Asu- 4) Mejoría espontánea del déficit neurológico. neurológica, destacando sólo una discreta rigi-
miendo que el paciente tiene un ictus, ¿cuál de las 5) TC cerebral normal. dez nucal. La presión arterial es de 15/8. Se rea-
siguientes causas es la única posible? liza TC craneal que resulta normal. ¿Cuál de las
Respuesta correcta: 4 siguientes opciones es la primera a seguir?
1) Disección de la arteria carótida interna izquier-
da. 1) Practicar un estudio angiográfico cerebral.
2) Trombosis de la arteria cerebelosa anteroinfe- P064 MIR 2007-2008 2) Realizar punción lumbar.
rior derecha. 3) Administrar una solución hiperosmolar para
3) Trombosis de la arteria cerebelosa posteroin- ¿Cuál de las siguientes lesiones vasculares ce- disminuir la presión intracraneal.
ferior derecha. rebrales conlleva un menor riesgo de sangrado 4) Observación clínica y repetición de la TC pasa-
4) Trombosis de la arteria vertebral izquierda. intracraneal? das 24 horas.
5) Trombosis de la arteria cerebelosa anteroinfe- 5) Disminuir farmacológicamente la presión arterial.
rior izquierda. 1) Angioma venoso.
2) Malformación arteriovenosa. Respuesta correcta: 2
Respuesta correcta: 4 3) Cavernoma.
4) Aneurisma micótico de la arteria cerebral media.
5) Aneurisma sacular de la arteria comunicante P241 MIR 2003-2004
P062 MIR 2008-2009 anterior.
Hombre de 85 años de edad, con antecedentes
En un paciente de 58 años sin antecedentes Respuesta correcta: 1 de hemorragia cerebral hace 2 años. Ingresa
de interés que acude a Urgencias con una he- por cuadro agudo de hemiparesia derecha y
miparesia derecha y afasia motora de noventa P117 MIR 2007-2008 somnolencia. En el TC urgente se objetiva un
minutos de evolución, que tiene una glucemia gran hematoma intracerebral lobar frontopa-
de 132, con una coagulación normal y una TC rietal izquierdo. El paciente no es hipertenso.
craneal sin hallazgos, usted indicaría: Hombre de 76 años con antecedentes de hi- ¿Cuál, entre las siguientes, es la etiología más
pertensión arterial en tratamiento con ena- probable de la hemorragia del paciente?
1) Anticoagulación con heparina sódica. lapril que consulta por presentar de forma
2) Anticoagulación con heparina de bajo peso brusca disartria y hemiparesia derecha que 1) Metástasis.
molecular. desaparece en doce horas. Eco-Doppler de 2) Aneurisma.
3) Fibrinólisis con rt-PA. troncos supraaórticos: ateromatosis carotídea 3) Traumatismo.
4) Antiagregación con aspirina. bilateral con estenosis del 55%), en la carótida 4) Tóxicos o medicamentos.
5) Antiagregación con clopidogrel. izquierda, y del 30% en la derecha. Electrocar- 5) Angiopatía amiloide.
diograma: fibrilación auricular con respuesta
Respuesta correcta: 3 ventricular a 90 Ipm. TC craneal: normal. Para la Respuesta correcta: 5

265
DESGLOSES MIR
P249 MIR 2003-2004 2) Disquinesias coreicas bajo tratamiento con le- P078 MIR 2011-2012
vodopa.
Los infartos lacunares suponen alrededor del 3) Distonía en el pie. Es patognomónico de la enfermedad llamada
20% de todos los accidentes cerebrovascu- 4) Alucinaciones visuales bajo tratamiento. GilIes de la Tourette:
lares. ¿Cuál de los siguientes enunciados le 5) Antecedentes familiares de parkinsonismo.
parece falso en relación con la localización de 1) La existencia de tics bucofonatorios.
dichos infartos lacunares? Respuesta correcta: 1 2) La existencia de tics motores.
3) Un nivel intelectual alto.
1) La hemiparesia motora pura se produce por 4) La existencia de ideas obsesivas.
un infarto en el brazo posterior de la cápsula P074 MIR 2012-2013 5) Los trastornos de conducta.
blanca interna.
2) Ictus sensitivo puro por un infarto de la por- En un paciente que se presenta en la segunda Respuesta correcta: 1
ción ventrolateral del tálamo. década de su vida con un cuadro progresivo de
3) La hemiparesia atáxica, por infarto en el cerebelo. parkinsonismo, temblor, distonía y alteración
4) La disartria y mano torpe, por infarto en la pro- de conducta, usted debe siempre realizar un P158 MIR 2010-2011
tuberancia. estudio para descartar la presencia de una en-
5) La disartria y mano torpe, por infarto en la rodi- fermedad cuyo tratamiento adecuado puede Un niño de 10 años traído a Urgencias porque
lla de la cápsula blanca interna. mejorar los síntomas neurológicos y detener desde hace 2 horas se le gira involuntariamen-
el curso clínico. ¿De qué enfermedad se trata? te el cuello a la derecha, asociando marcado
Respuesta correcta: 3 dolor cervical en cada giro. La abuela, poste-
1) Enfermedad de Huntington. riormente, nos dice que está vomitando desde
2) Enfermedad de Wilson. ayer, por lo que le dio un jarabe; ¿cuál sería la
3) Síndrome de Tourette. actitud terapéutica más acertada?
4) Corea de Sydenham.
T05 Trastornos
del movimiento 5) Enfermedad de Parkinson de inicio juvenil. 1) Inyectar un anticolinérgico intravenoso.
2) No tratar hasta no realizar en los días próximos
Respuesta correcta: 2 un electroencefalograma.
P147 MIR 2013-2014 3) No tratar hasta no saber en días próximos el re-
sultado del cultivo del líquido cefalorraquídeo.
Los síntomas motores en la enfermedad de P075 MIR 2012-2013 4) Llamar al psiquiatra de urgencias.
Parkinson predominan y definen esta en- 5) Poner un antibiótico intravenoso tras obtener
tidad. No obstante, los síntomas en otras Mujer de 75 años que consulta por insomnio. muestra de exudado faríngeo.
esferas, son a veces muy relevantes y se de- Refiere que desde hace años se despierta por
nominan con el nombre general de “Mani- la noche con sensación de hormigueo, que- Respuesta correcta: 1
festaciones no motoras de la enfermedad de mazón en las piernas y a veces en los brazos;
Parkinson”. ¿Cuál de los siguientes se conside- estos síntomas se presentan también por la
ra un síntoma no motor de la enfermedad de tarde. Nota mejoría al mover las piernas, pero P069 MIR 2009-2010
Parkinson? los síntomas recurren durante el reposo, por lo
que no puede volver a conciliar el sueño. Este Nos consulta un hombre de 49 años, diestro, por
1) Hipoglucemia. cuadro nos podría orientar al diagnóstico de temblor y torpeza en su mano derecha de 3 me-
2) Hipotensión ortostática. un síndrome de piernas inquietas. ¿Cuál de las ses de evolución. Salvo una hipercolesterolemia
3) Crisis de ausencia. siguientes es FALSA? que trata con dieta, no tiene otros antecedentes
4) Cefalea. relevantes. En la exploración del miembro supe-
5) Polineuropatía motora. 1) El diagnóstico de esta afectación se basa en rior derecho se objetiva temblor de reposo que
criterios clínicos. aparece con la distracción, rigidez espontánea
Respuesta correcta: 2 2) Habría que realizar una analítica básica que in- en rueda dentada y una marcada bradicinesia.
cluyera perfil férrico, hormonas tiroideas y B12 En el miembro inferior derecho se observa una
y ácido fólico. mínima pero evidente afectación del taconeo.
P072 MIR 2012-2013 3) El tratamiento estaría indicado en pacientes que No tiene alteraciones semiológicas en los miem-
presentan alteración del sueño o de la calidad de bros izquierdos y los reflejos posturales son nor-
En un paciente con parkinsonismo, ¿cuál de vida, pero no altera el curso de la enfermedad. males. El paciente comenta espontáneamente
las siguientes situaciones le parece MENOS 4) El diagnóstico se confirma con biopsia muscular. que los síntomas han comenzado a afectar su
probable que ocurra en la enfermedad de Par- 5) El pramipexol y el ropirinol se utilizan en el tra- vida personal y laboral. ¿Cuál de las siguientes
kinson? tamiento. opciones terapéuticas es la más indicada en
este caso, especialmente si queremos reducir el
1) Ausencia de respuesta a la levodopa. Respuesta correcta: 4 riesgo de disquinesias o fluctuaciones motoras?

NEUROLOGÍA 266
Y NEUROCIRUGÍA
Neurología y neurocirugía
1) Agonista de la dopamina. P058 MIR 2005-2006 P242 MIR 2003-2004
2) Levodopa/carbidopa de liberación estándar.
3) Levodopa/carbidopa de liberación “retard”. Debe dudar de un diagnóstico de enferme- Un paciente de 60 años refiere que desde hace
4) Levodopa/carbidopa estándar + estacapona. dad de Parkinson si en la exploración se en- años le tiemblan las manos al sostener la cu-
5) Levodopa/carbidopa estándar + rasagilina. cuentra: chara, el vaso o el bolígrafo, sobre todo si está
nervioso o fatigado, y estos síntomas mejoran
Respuesta correcta: 1 1) Temblor de reposo. con pequeñas cantidades de vino. Su padre,
2) Temblor postural. ya fallecido, había presentado temblor en las
3) Parpadeo aumentado. manos y la cabeza. La exploración neurológica
P057 MIR 2008-2009 4) Micrografía. sólo muestra temblor de actitud simétrico en
5) Reflejos de estiramiento muscular hiperacti- ambas manos. Este cuadro clínico es probable-
En un paciente diagnosticado de enfermedad vos. mente consecuencia de:
de Parkinson hace un año, todos los siguientes
síntomas nos harían dudar del diagnóstico EX- Respuesta correcta: ANU 1) Un hipotiroidismo familiar.
CEPTO uno de ellos: 2) Una enfermedad de Parkinson incipiente.
3) Síntomas de deprivación etílica.
1) Ausencia de respuesta a L-dopa. P058 MIR 2004-2005 4) Un temblor esencial.
2) Asimetría de los síntomas. 5) Una neurosis de ansiedad orgánica familiar.
3) Incontinencia urinaria u ortostatismo frecuentes. El diagnóstico de la enfermedad de Parkison
4) Inestabilidad postural con caídas frecuentes. es fundamentalmente clínico. ¿Cuál de los si- Respuesta correcta: 4
5) Dificultad para la mirada hacia abajo. guientes hallazgos es muy improbable en esta
enfermedad y cuestiona seriamente su diag-
Respuesta correcta: 2 nóstico? P243 MIR 2003-2004

1) Seborrea. ¿Cuál de los siguientes hallazgos es muy fre-


P057 MIR 2006-2007 2) Torpeza en los movimientos alternantes. cuente en la enfermedad de Parkinson idiopá-
3) Depresión. tica?
¿Cuál de los siguientes rasgos es característico 4) Limitación en los movimientos de la mirada
de los signos extrapiramidales de la enferme- hacia abajo. 1) Demencia al inicio.
dad de Parkinson? 5) Micrografía. 2) Mioclonías.
3) Blefaroespasmo.
1) La escasa o nula respuesta a la levodopa en fa- Respuesta correcta: 4 4) Retrocollis.
ses incipientes. 5) Pérdida de movimientos asociados en la marcha.
2) La instauración relativamente rápida.
3) La asimetría. P239 MIR 2003-2004 Respuesta correcta: 5
4) Su asociación con temblor cefálico.
5) Su asociación con piramidalismo. El síndrome de las piernas inquietas es un tras-
torno crónico frecuente. En relación al mismo,
Respuesta correcta: 3 indique la respuesta correcta entre las que a
continuación se enumeran: T06 Enfermedades por
alteración de la mielina
P064 MIR 2006-2007 1) Con frecuencia se asocia a una defi ciencia de
hierro. P150 MIR 2013-2014
Una de las siguientes enfermedades no suele 2) Es un trastorno de origen psicógeno en la ma-
cursar con parkinsonismo o síndrome rígido- yoría de los casos. Señale en qué situación clínica NO indicaría
acinético, y no entra habitualmente en el 3) Es una manifestación de la polineurapatía sen- interferón beta de inicio en un paciente diag-
diagnóstico de la enfermedad de Parkinson sitiva de fi bra pequeña. nosticado de esclerosis múltiple:
diopática: 4) La existencia de movimientos periódicos en las
piernas durante el sueño es condición necesa- 1) Múltiples brotes recurrentes.
1) Parálisis supranuclear progresiva. ria para el diagnóstico. 2) Mala respuesta a los corticoides en los brotes.
2) Hidrocefalia a presión normal. 5) Los síntomas ocurren cuando el paciente in- 3) Haber padecido un solo brote de la enferme-
3) Enfermedad de cuerpos de Lewy. tenta dormir en postura incómoda (por ejem- dad con secuelas.
4) Enfermedad de Wilson. plo, sentado) pero no en una cama conforta- 4) Tener más de 50 años.
5) Síndrome de Gilles de la Tourette. ble. 5) Una forma clínica primaria progresiva.

Respuesta correcta: 5 Respuesta correcta: 1 Respuesta correcta: 5

267
DESGLOSES MIR
P153 MIR 2013-2014 Después de llegar al diagnóstico definitivo del 3) Lupus eritematoso sistémico con afectación
paciente, señale cuál es el tratamiento más cerebral.
¿Qué fármaco debe recomendarse para el tra- apropiado en la actualidad para el enfermo: 4) Patología vascular protrombótica.
tamiento de la epilepsia mioclónica juvenil? 5) Migraña con aura visual.
1) Inmunoglobulinas intravenosas.
1) Fenobarbital. 2) Vitamina B12 parenteral. Respuesta correcta: 2
2) Ácido valproico. 3) Interferón beta.
3) Oxcarbacepina. 4) Tratamiento sintomático de la espasticidad y
4) Difenilhidantoína. de la vejiga neurógena. P056 MIR 2008-2009
5) Topiramato. 5) Corticoides intravenosos.
Varón de 18 años que es traído a Urgencias
Respuesta correcta: 2 Respuesta correcta: 4 por la familia por presentar, mientras estaba
comiendo, desviación ocular y cefálica a la
derecha, seguido de pérdida de conocimien-
P070 MIR 2012-2013 P063 MIR 2010-2011 to brusca, durante la cual se evidenciaron
movimientos tónico clónicos de los cuatro
En un paciente diagnosticado de esclerosis múl- ¿Cuál de los siguientes subtipos celulares NO miembros, de unos 4 minutos de duración,
tiple podemos encontrar todos los siguientes ha- participa en la inmunopatología de la esclero- quedando posteriormente en sueño profundo
llazgos en el líquido cefalorraquídeo, EXCEPTO: sis múltiple? durante unas dos horas. ¿Cuál es el diagnóstico
de sospecha?
1) Leve elevación de proteínas. 1) Linfocito T CD4+.
2) Aumento de inmunoglobulinas tipo IgM e IgG. 2) Linfocito T CD8+. 1) Síncope vasovagal.
3) Bandas oligoclonales en el LCR y no en el suero. 3) Macrófagos. 2) Crisis mioclónica.
4) Número de linfocitos hasta 100-200 por ml. 4) Linfocitos B. 3) Crisis tónico clónica generalizada.
5) LCR bioquímicamente normal. 5) Basófilos. 4) Crisis parcial secundariamente generalizada.
5) Crisis parcial compleja.
Respuesta correcta: 4 Respuesta correcta: 5
Respuesta correcta: 4

P017 MIR 2011-2012 P064 MIR 2009-2010


P058 MIR 2008-2009
Pregunta vinculada a la imagen n.º 9 ¿Cuál de los siguientes signos o síntomas apa-
rece raramente en la esclerosis múltiple? Una mujer de 32 años acude a un Servicio de
Hombre de 43 años que consulta por un cuadro Urgencias por un cuadro de pérdida de fuer-
de debilidad de miembro inferior izquierdo al ca- 1) Síndrome piramidal. za en ambas piernas y dificultad para orinar
minar y urgencia urinaria. Empeoramiento en los 2) Hemianopsia homónima. de instauración en 48 horas. En la explora-
dos últimos años. En la exploración se objetiva una 3) Neuritis óptica. ción se aprecia un nivel sensitivo asociado
paraparesia espástica asimétrica de predominio 4) Ataxia y temblor cerebelosos. a una hiperreflexia difusa con signo de Ba-
izquierdo y una alteración de la sensibilidad vibra- 5) Diplopía. binski bilateral. Un año antes fue evaluada
toria de ambos miembros inferiores. Se realiza una en oftalmología por una pérdida de agudeza
resonancia magnética cerebral y cervical. ¿Cuál es Respuesta correcta: 2 visual en el ojo derecho que recuperó com-
el diagnóstico más probable del paciente? pletamente en 6 semanas. En la resonancia
magnética se aprecian numerosas lesiones
1) Mielopatía vascular. P228 MIR 2009-2010 supra e infratentoriales. ¿Cuál de los siguien-
2) Esclerosis múltiple primaria progresiva. tes tratamientos NO modificará el curso de su
3) Mielopatía por déficit de vitamina B12. Mujer de 26 años de edad con antecedentes de enfermedad?
4) Mielopatía en el contexto de una colagenosis. un cuadro de alteración sensitiva de la pierna
5) Neuromielitis óptica (síndrome de Devic). izquierda hace 6 meses, que acude a Urgencias 1) Interferón alfa.
por un cuadro de dolor ocular a la movilización 2) Acetato de glatiramer.
Respuesta correcta: 2 del ojo derecho y disminución de la agudeza 3) Interferón beta 1-a.
visual de 5 días de duración. ¿Cuál será la pri- 4) Mitoxandrona.
mera sospecha diagnóstica? 5) Interferón beta 1-b.
P018 MIR 2011-2012
1) Vasculitis en el marco de una colagenosis. Respuesta correcta: 1
Pregunta vinculada a la imagen n.º 9 2) Esclerosis múltiple.

NEUROLOGÍA 268
Y NEUROCIRUGÍA
Neurología y neurocirugía
P061 MIR 2007-2008 P056 MIR 2005-2006 P244 MIR 2003-2004

La esclerosis múltiple es un proceso inflamato- Mujer de 24 años diagnosticada de esclerosis Un hombre de 28 años acude a consulta refi-
rio y desmielinizante del SNC. En el diagnósti- múltiple hace 2 años, después de sufrir un bro- riendo desde hace 10 días un cuadro de alte-
co de esta enfermedad es muy útil la presencia te inicial de neuritis óptica retrobulbar. Señale ración de la sensibilidad de hemicuerpo que
de: cuál es el síntoma o el signo que NO esperaría incluye la cara. Tiene como antecedentes haber
encontrar en la paciente: padecido una visión borrosa por el ojo izquier-
1) Elevaciones del ácido úrico en plasma. do hace un año, que recuperó por completo
2) Bandas oligoclonales en el líquido cefalorra- 1) Disartria. en un mes. En la exploración actual se objetiva
quídeo. 2) Incontinencia urinaria. una hemihipoestesia izquierda con signo de
3) Más de 100 linfocitos por microlitro en el líqui- 3) Fatiga crónica. Babinski de ese lado. ¿Qué prueba diagnóstica
do cefalorraquídeo. 4) Afasia. es la más apropiada para conocer la etiología
4) Cifras elevadas de ácidos grasos de cadena 5) Importante sensibilidad anómala al calor. más frecuente de este proceso?
muy larga en plasma.
5) Más de 50 polimorfonucleares por microlitro Respuesta correcta: 4 1) TC cerebral con contraste.
en el líquido cefalorraquídeo. 2) Estudio rutinario del LCR.
3) Estudios serológicos de virus.
Respuesta correcta: 2 P056 MIR 2004-2005 4) Resonancia magnética cerebral.
5) Potenciales evocados visuales.
Mujer de 32 años que acude a la consulta por
P059 MIR 2006-2007 síntomas agudos compatibles con una oftal- Respuesta correcta: 4
moplejia internuclear bilateral. Ante la sospe-
Mujer de 40 años diagnosticada de esclerosis cha de una esclerosis múltiple, ¿qué resultado
múltiple hace 15 años y que continúa presen- de qué prueba diagnóstica complementaria
tando brotes de su enfermedad con secuelas
progresivas neurológicas. ¿Qué tratamiento en
NO esperaría encontrar?
T07 Epilepsia
la actualidad NO estaría indicado en la pacien- 1) Potenciales evocados visuales alterados en el
te por falta de eficacia? ojo izquierdo. P071 MIR 2012-2013
2) Presencia de bandas oligoclonales en el LCR y
1) Ciclosporina. no en el suero. Niño de 10 años con episodios breves de dis-
2) Mitoxantrona. 3) Anticuerpos antinucleares negativos en sue- tracciones (< 1 minuto) en los que no responde
3) Interferón beta 1-a. ro. a llamadas y parpadea. Un EEG muestra descar-
4) Rehabilitación. 4) Múltiples alteraciones de señal en cerebro y gas de punta-onda a 3 ciclos por segundo. El tra-
5) Interferón beta 1-b. médula espinal en resonancia magnética. tamiento electivo de primera línea lo haría con:
5) LCR con 120 células por microlitro.
Respuesta correcta: 1 1) Valproato.
Respuesta correcta: 5 2) Carbamacepina.
3) Fenitoína.
P055 MIR 2005-2006 4) Gabapentina.
P240 MIR 2003-2004 5) Clonazepam.
Hombre de 32 años que hace 2 años fue diag-
nosticado de esclerosis múltiple. Acude a Ur- Señale cuál es la indicación de tratamiento con Respuesta correcta: 1
gencias porque desde hace 4 días presenta interferón beta en pacientes con esclerosis
paraparesia severa y progresiva que le dificulta múltiple en la actualidad:
el caminar. ¿Qué actitud terapéutica adoptaría P224 MIR 2012-2013
en ese momento? 1) Tratamiento sintomático de los brotes.
2) Prevención de los brotes en pacientes con for- Todos los anticonvulsivantes que aparecen a
1) Inmunoglobulinas intravenosas. mas clínicas recurrentes-remitentes. continuación, EXCEPTO uno que debe señalar,
2) Pulsos intravenosos de 1 gramo de metilpred- 3) Tratamiento de la discapacidad de las formas bloquean los canales de sodio dependientes
nisolona. primarias progresivas. de voltaje:
3) Plasmaféresis. 4) Tratamiento para las neuritis ópticas.
4) Rehabilitación motora intensa. 5) Tratamiento de todas las formas clínicas, pero 1) Lamotrigina.
5) Copolímero I. sólo en casos muy avanzados (sin deambular). 2) Carbamazepina.
3) Gabapentina.
Respuesta correcta: 2 Respuesta correcta: 2 4) Fenitoína.

269
DESGLOSES MIR
5) Ácido valproico. 2) Holter ECG para diagnóstico de cardiopatía P056 MIR 2008-2009
arrítmica.
Respuesta correcta: 3 3) EEG de rutina para diagnóstico del tipo de epi- Varón de 18 años que es traído a Urgencias por la
lepsia (generalizada o focal). familia por presentar, mientras estaba comien-
4) Resonancia magnética cerebral para detectar do, desviación ocular y cefálica a la derecha,
P082 MIR 2011-2012 lesiones epileptógenas (displasia cortical, tu- seguido de pérdida de conocimiento brusca,
mor, esclerosis temporal media). durante la cual se evidenciaron movimientos tó-
Paciente, mujer de 18 años, con historia de au- 5) Determinación de glucemia capilar para diag- nico clónicos de los cuatro miembros, de unos 4
sencias entre los 6-9 años, crisis tónico-clónicas nóstico de hipoglucemia. minutos de duración, quedando posteriormen-
generalizadas de reciente comienzo y saltos vio- te en sueño profundo durante unas dos horas.
lentos de miembros superiores al desayunar. La Respuesta correcta: 1 ¿Cuál es el diagnóstico de sospecha?
clínica empeora con salidas nocturnas de fines
de semana. Un EEG muestra descargas de poli- 1) Síncope vasovagal.
puntas agudas a 6 ciclos/segundo. El diagnósti- P068 MIR 2009-2010 2) Crisis mioclónica.
co más probable es: 3) Crisis tónico clónica generalizada.
Una mujer de 23 años es traída a Urgencias 4) Crisis parcial secundariamente generalizada.
1) Gran mal epiléptico. tras sufrir su primer episodio de pérdida de 5) Crisis parcial compleja.
2) Síndrome de Lennox-Gastaut. consciencia. La paciente sólo recuerda que
3) Epilepsia sintomática por esclerosis temporal estaba caminando y comenzó a sentir náu- Respuesta correcta: 4
mesial. seas, sudoración, sensación de calor ascen-
4) Pequeño mal atípico. dente y oscurecimiento visual. Un testigo
5) Epilepsia mioclónica juvenil. afirmó que durante el episodio, que apenas P055 MIR 2007-2008
duró 10 segundos, tuvo sacudidas en los
Respuesta correcta: 5 brazos y se orinó. La paciente se recuperó En un periodo de 3 meses, un hombre de 42
rápidamente. ¿Cuál es el diagnóstico más años ha presentado cuatro episodios de des-
probable? conexión de su entorno, asociados con postura
P202 MIR 2011-2012 distónica de la mano derecha y movimientos
1) Crisis epiléptica parcial compleja. de masticación, de un minuto de duración,
¿Cuál de los siguientes antiepilépticos se espe- 2) Síncope. quedando a continuación confuso y con di-
ra que produzca un menor número de interac- 3) Histeria. ficultad para la expresión verbal durante 10
ciones farmacológicas? 4) Crisis epiléptica primariamente generalizada. minutos. Después se recupera con normali-
5) Crisis epiléptica parcial, secundariamente ge- dad pero no recuerda lo que le ha ocurrido.
1) Fenitoína. neralizada. ¿Cuál es la actitud más correcta?
2) Ácido valproico.
3) Levetiracetam. Respuesta correcta: 2 1) Iniciar tratamiento con clobazam oral y realizar
4) Carbamazepina. un electroencefalograma (EEG).
5) Oxcarbazepina. 2) Iniciar tratamiento con carbamacepina y reali-
P168 MIR 2009-2010 zar una resonancia magnética (RM) y un EEG.
Respuesta correcta: 3 3) Realizar un EEG y una RM de forma ambula-
Nos llega a la consulta un niño de 7 años diag- toria e iniciar tratamiento con fenitoína intra-
nosticado de crisis de ausencias típicas. ¿Qué venosa.
P064 MIR 2010-2011 dato clínico de los abajo expuestos NO espera- 4) Ingresar en la unidad de cuidados intensivos e
ría encontrar en el paciente? iniciar tratamiento con fenitoína ultravenosa.
En un paciente diagnosticado de epilepsia que 5) Recomendar observación por su familia y vol-
presenta episodios de falta de respuesta a es- 1) Automatismos motores leves en la cara duran- ver a revisión en tres meses.
tímulos externos, movimientos irregulares de te la crisis.
las cuatro extremidades, ojos cerrados, emi- 2) Confusión poscrítica. Respuesta correcta: 2
sión de llanto y movimientos pélvicos, de cinco 3) Descargas generalizadas punta-onda a 3 Hz en
a veinte minutos de duración y que no respon- el electroencefalograma durante la crisis.
den al tratamiento con fármacos antiepilépti- 4) Pérdida completa de la conciencia de segun- P056 MIR 2007-2008
cos. ¿Cuál es el estudio complementario con dos de duración.
mayor probabilidad de aclarar el diagnóstico? 5) Con maniobras de hiperventilación podemos Una joven de 13 años sin antecedentes personales
provocar las crisis del niño. de interés, presenta bruscamente mientras pasea
1) Monitorización vídeo-EEG para diagnóstico de con sus padres por un centro comercial, cuadro
pseudocrisis (crisis psicógenas). Respuesta correcta: 2 vertiginoso asociado a vómitos. Inmediatamente

NEUROLOGÍA 270
Y NEUROCIRUGÍA
Neurología y neurocirugía
después, refiere parestesias en hemicuerpo dere- 1) Carbamazepina. 5) Ausencia atípica.
cho y disartria, que remiten en pocos minutos, y a 2) Fenitoína.
continuación cefalea, que mantiene a su llegada 3) Etosuximida. Respuesta correcta: 3
al centro de salud. Señale la respuesta correcta: 4) Valproato sódico.
5) Clonazepam.
1) El diagnóstico más probable es el de síncope P061 MIR 2004-2005
vasovagal. Respuesta correcta: 2
2) Si la exploración física es normal no se requie- Un paciente de 40 años, sin antecedentes rele-
ren más estudios. vantes, es traído a Urgencias por haber presen-
3) La ausencia de antecedentes personales exclu- P053 MIR 2006-2007 tado desviación de la cabeza hacia la izquierda,
ye el diagnóstico de migraña. convulsiones que se iniciaron en miembros
4) El empleo de analgésicos y antieméticos no Mujer de 18 años con antecedentes de menin- izquierdos y se generalizaron enseguida a los
son aconsejables en este supuesto. gitis al año de vida que cursó con crisis convul- cuatro miembros, con pérdida de conciencia,
5) Se debe evaluar la existencia de estímulos des- sivas; a la edad de 16 años empieza a presentar incontinencia vesical y estado confusional de
encadenantes. episodios de presentación plurimensual de una media hora de duración. Independien-
desconexión del medio precedidos de una temente de los hallazgos de la exploración
Respuesta correcta: 5 sensación epigástrica ascendente. Durante la clínica y la analítica clínica de rutina, debería
fase de desconexión realiza movimientos au- realizarse con premura como primera medida:
tomáticos con la boca y con miembro superior
P184 MIR 2007-2008 derecho y adopta una actitud distónica del 1) TC cerebral.
miembro superior izquierdo, respondiendo 2) Determinación de alcoholemia.
Niña de 2 años que, estando previamente bien, de forma incoherente a las preguntas. Estos 3) Determinación de opiáceos en sangre y orina.
sufre episodio brusco de desconexión del me- episodios duran aproximadamente un minuto 4) Electroencefalograma.
dio e hipertonía, con estridor y sialorrea, du- y posteriormente presenta hipersomnia. Indi- 5) Punción lumbar.
rante aproximadamente 2 minutos, quedando que la
porteriormente somnolienta durante aproxi- Respuesta correcta: 1
madamente 5 minutos. A su llegada al centro 1) El paciente presenta crisis parciales complejas
de salud se objetiva temperatura axilar de 38,9 con semiología automotora características del
ºC. En relación con esta niña, ¿cuál de las si- lóbulo temporal medial. P248 MIR 2003-2004
guientes afirmaciones es correcta? 2) Los cuadros descritos son muy sugestivos
de crisis parciales complejas de origen fron- En relación con la epilepsia es FALSO que:
1) Presenta un proceso benigno pero que casi tal.
siempre ocurre. 3) El paciente presenta un síndrome de ausencias 1) En la esclerosis mesial del lóbulo temporal sue-
2) Debe remitirse a un centro hospitalario para juveniles. le haber antecedentes de crisis febriles.
realización de una prueba de neuroimagen. 4) Las crisis son las características de una epilep- 2) En la fase tónica de una crisis generalizada to-
3) Si el foco causal de la fiebre es claro y banal sia mioclónica juvenil. nicoclónica hay cianosis y midriasis.
y la recuperación clínica completa, puede 5) El paciente presenta crisis parciales complejas 3) Las crisis febriles suelen aparecer entre los tres
manejarse con antitérmicos y observación con semiología automotora características del meses y los cinco años de edad.
domiciliaria sin realizar ninguna prueba com- lóbulo temporal lateral. 4) Las crisis de ausencia típica se relacionan con
plementaria. patología del lóbulo temporal.
4) Será necesaria la realización de un EEG más Respuesta correcta: 1 5) Los accidentes cerebrovasculares son una cau-
adelante. sa frecuente de crisis en los ancianos.
5) Presenta un riesgo mayor que el resto de la po-
blación de desarollo posterior de alguna forma P061 MIR 2005-2006 Respuesta correcta: 4
de epilepsia.
Un episodio caracterizado por sensación epi-
Respuesta correcta: 3 gástrica que asciende hacia el tórax, seguido
por dificultad para conectar con el entorno, Enfermedades
P222 MIR 2007-2008
movimientos de masticación, distonia de una T08 degenerativas del
sistema nervioso
mano y falta de respuesta, de un minuto de
duración, con amnesia postcrítica, es una crisis:
¿Cuál de los siguientes antiepilépticos admi- P061 MIR 2008-2009
nistrado en mujeres embarazadas puede pro- 1) Parcial simple.
ducir, en el recién nacido, una deficiencia de 2) Parcial secundariamente generalizada. Mujer de 60 años que consulta por cuadro de 4
factores de la coagulación dependientes de 3) Parcial compleja. meses de evolución de pérdida de fuerza pro-
vitamina K? 4) Ausencia típica. gresiva en miembros inferiores y disfonía. En

271
DESGLOSES MIR
la exploración realizada no presentaba fascicu- lidad en la pierna derecha. En la exploración 2) La detección de antígeno de herpes simple 1
laciones, los pares craneales estaban conser- se objetiva una paresia con amiotrofia de y 2 en el LCR.
vados, no presentaba déficit campimétricos, el miembro inferior derecho y una hiperreflexia 3) Detección de IgG frente al virus del herpes sim-
balance muscular en miembros superiores es- miotática de dicho miembro. ¿Cuál es su ple 1 y 2 en el LCR por prueba de ELISA.
taba conservado aunque presentaba amiotro- diagnóstico? 4) Detección de antígeno de los virus del herpes
fia en primer interóseo izquierdo. En miembros simple 1 y 2 en orina.
inferiores presentaba paresia próximal bilateral 1) Hernia discal lumbar deficitaria. 5) Prueba de la reacción en cadena de la polime-
asimétrica, estando el miembro inferior dere- 2) Síndrome de Guillain-Barré. rasa con cebadores específicos para los virus del
cho a 4/5 y miembro inferior izquierdo a 3+/5; 3) Esclerosis lateral amiotrófica. herpes simple 1 y 2 en una biopsia cerebral o
a nivel distal presentaba una paresia del grupo 4) Neuropatía por enfermedad de Lyme. alternativamente en el líquido cefalorraquídeo.
anteroexterno derecho a 4/5 siendo el resto del 5) Esclerosis múltiple.
balance normal; las sensibilidades superficial Respuesta correcta: 5
y profunda estaban conservadas, los reflejos Respuesta correcta: 3
osteotendinosos estaban exaltados en los 4
miembros con aumento de área reflexógena P058 MIR 2006-2007
siendo la respuesta plantar izquierda extensora
y la derecha flexora. Un estudio de resonancia Enfermedades virales Hombre de 30 años ex-adicto a drogas vía pa-
nuclear magnética cerebral y de columna com- T09 y priónicas del sistema
nervioso
renteral y sin otros antecedentes que consulta
pleta de bajo campo fue normal. ¿Cuál sería la por cuadro de curso progresivo y subagudo
actitud más lógica a realizar de inicio? de deterioro cognitivo, presentando en el es-
P062 MIR 2009-2010 tudio de resonancia magnética lesiones mul-
1) Solicitar un estudio electromiográfico con esti- tifocales en sustancia blanca frontal izquierda,
mulación repetiva por la alta sospecha de en- ¿En cuál de las siguientes enfermedades se ob- periventricular derecha y occipital derecha no
fermedad de placa neuromuscular. serva pleocitosis en el LCR? captantes de contraste. Indique la afirmación
2) Solicitar una bioquímica urgente ante la sospe- correcta:
cha de una parálisis hipopotasémica. 1) Esclerosis lateral amiotrófica.
3) Solicitar una arteriografía de columna cervical 2) Síndrome de Guillain-Barré. 1) Se debería realizar un estudio electroencefalo-
ante la sospecha de una malformación arterio- 3) Encefalitis herpética. gráfico y una determinación en LCR de proteí-
venosa cervical. 4) Leucoencefalopatía mutifocal progresiva. na 14-3-3 para descartar una enfermedad de
4) Realizar un estudio electromiográfico con es- 5) Enfermedad de Creutzfeld-Jakob. Creutzfeldt-Jakob.
tudio de unidades motoras. 2) Estaría indicada una punción lumbar solicitan-
5) Determinación de anticuerpos antiGMl ante la Respuesta correcta: 3 do serología en líquido cefalorraquídeo para
alta sospecha de una neuropatía motora mul- toxoplasma.
tifocal con bloqueos de conducción. 3) Se debería realizar serología VIH y punción
P114 MIR 2009-2010 lumbar para realización de PCR (reacción en
Respuesta correcta: 4 cadena de la polimerasa) de papovavirus JC.
¿Qué es cierto de la encefalitis herpética? 4) Lo más probable es una meningitis tuberculo-
sa y se debería iniciar tratamiento urgente con
P234 MIR 2005-2006 1) Está causada por el virus del herpes tipo II. rifampicina, piracinamida y etambutol.
2) Afecta con preferencia los lóbulos occipitales. 5) Es poco probable que se trate de una leucoen-
En la esclerosis lateral amiotrófica, la lesión se 3) Causa un LCR purulento. cefalopatía multifocal progresiva al no captar
localiza en: 4) El tratamiento de elección es aciclovir. contraste las lesiones.
5) La resonancia magnética es diagnóstica.
1) Corteza entorrinal. Respuesta correcta: 3
2) Núcleo caudado. Respuesta correcta: 4
3) Sustancia negra.
4) Corteza cerebelosa. P232 MIR 2004-2005
5) Asta anterior de la médula. P208 MIR 2009-2010
La tríada: alteración espongiforme de las neu-
Respuesta correcta: 5 La prueba de elección para establecer el diag- ronas con aspecto microvacuolado de la sus-
nóstico de encefalitis herpética (causada por los tancia gris, astrogliosis y placas amiloides, se
virus del herpes simple tipo 1 o tipo 2) en un pa- presenta en una de las siguientes entidades:
P055 MIR 2004-2005 ciente con sospecha de dicho proceso, es:
1) Enfermedad de Parkinson.
Mujer de 64 años que consulta por clínica 1) El cultivo del líquido cefalorraquídeo en una 2) Enfermedad de Alzheimer.
progresiva en los últimos 4 meses de debi- línea celular. 3) Enfermedad de Pick.

NEUROLOGÍA 272
Y NEUROCIRUGÍA
Neurología y neurocirugía
4) Esclerosis múltiple. torpe y una desviación en los ojos divergente que 3) Al tratarse de una enfermedad de herencia au-
5) Enfermedad de Creutzfeldt-Jakob. antes no tenía. Según parece, ha estado bebien- tosómica dominante, el riesgo de tener un hijo
do alcohol hasta unas horas antes de llevarlo a afecto es del 50%.
Respuesta correcta: 5 urgencias. Probablemente se trate de un caso de: 4) El gen MFN2 está en el cromosoma 1, por lo que
todos los hijos varones están exentos de riesgo,
1) Síndrome de abstinencia complicado. en cambio todas sus hijas serán portadoras.
2) Enfermedad de Korsakoff. 5) No hay riesgo, puesto que la enfermedad no
Enfermedades 3) Encefalopatía de Wernicke. ha aparecido en el padre (penetrancia incom-

T10 nutricionales y
metabólicas del
4)
5)
Alucinosis alcohólica.
Intoxicación etílica aguda.
pleta).

sistema nervioso Respuesta correcta: 3


Respuesta correcta: 3
P053 MIR 2008-2009
P057 MIR 2007-2008
Un paciente es llevado a Urgencias por haber
sido hallado desorientado en la calle. Tiene
alrededor de 60 años y aspecto descuidado.
T11 Neuropatías En un enfermo con una clínica de pérdida de
fuerza progresiva de 48 horas de evolución
Huele a alcohol. La exploración física revela un que se inició a nivel distal y ha ido ascendiendo
paciente desorientado, confuso y apático. Está P144 MIR 2013-2014 progresivamente, una de las siguientes afirma-
levemente atáxico y presenta múltiples telan- ciones es FALSA:
giectasias cutáneas. TA: 140/85 mmHg; pulso ¿Qué signo o síntoma NO es típico del síndro-
84 lpm y frecuencia respiratoria 16 rpm. ¿Cuál me de Guillain-Barré? 1) Sospecharía un síndrome de Guillain-Barré.
de las siguientes medidas sería INCORRECTA en 2) Vigilaría la aparición de complicaciones por
una evaluación inicial en la visita de Urgencias? 1) Parálisis facial. neuropatía autónoma.
2) Alteración de esfínteres. 3) Esperaría encontrar disociación albúminacito-
1) Punción lumbar tras realización de fondo de ojo. 3) Hipo o arreflexia tendinosa. lógica a partir de los 10 días desde el inicio del
2) TC craneal. 4) Debilidad muscular. cuadro.
3) Hemograma completo y bioquímica básica. 5) Disautonomía. 4) Indicaría tratamiento con inmunoglobulinas
4) Administración de glucosado al 5% i.v. y poste- intravenosas.
riormente tiamina i.v. Respuesta correcta: 2 5) Los hallazgos neurofisiológicos aparecen antes
5) Determinación de tóxicos en sangre y orina. de que sea evidente la clínica.

Respuesta correcta: 4 P211 MIR 2010-2011 Respuesta correcta: 5

Un hombre de 21 años es portador de una


P054 MIR 2007-2008 mutación (de etiología validada) en el gen P052 MIR 2006-2007
MFN 2 (mitofusin 2 cromosoma 1p36.2). Re-
En un paciente que presenta un cuadro agudo cientes estudios sitúan tales mutaciones como Hombre de 24 años con antecedentes de gas-
confusional, oftalmoparesia por afectación del sex- la causa más frecuente de la enfermedad de troenteritis aguda tres semanas antes, que
to par bilateral y ataxia de la marcha, pensaría en: Charcot Marie-Tooth tipo 2A (CMT2A). El gen consulta por pérdida progresiva de fuerza en 4
MFN2 determina la fusión entre las mitocon- miembros de 3 días de evolución, habiéndose
1) Encefalopatía hiperglucémica. drias, circunstancia importante de su fisiología iniciado a nivel distal en miembros inferiores
2) Encefalopatía de Korsakoff. y morfología. Acude a la consulta de consejo asociando parestesias en manos y pies. A la ex-
3) Infarto cerebeloso. genético con su esposa en la que un reciente ploración presentaba tetraparesia de predomi-
4) Administrarle inmediatamente tiamina. análisis ha descartado mutaciones en el citado nio en miembros inferiores, arreflexia universal
5) Intoxicación por plomo. gen MFN2. Ambos están asintomáticos. ¿Cuál y ausencia de signos piramidales. ¿Qué resulta-
es el riesgo de transmisión de la enfermedad? dos NO esperaría encontrar en el estudio diag-
Respuesta correcta: 4 nóstico realizado a este paciente?
1) No hay riesgo, porque el CMT2A es mayorita-
riamente de herencia recesiva y ambos padres 1) Estudio electroneurográfico compatible con
P165 MIR 2007-2008 han de ser portadores (la esposa no lo es) para polineuropatía desmielinizante.
poder transmitirla. 2) Bioquímica de líquido cefalorraquídeo con 250
Un paciente bebedor habitual de alcohol es lle- 2) Al tratarse de un efecto sobre las mitocondrias, células con 90% de mononucleares e hiper-
vado a Urgencias con un cuadro de desorienta- la enfermedad únicamente se transmitirá vía proteinorraquia.
ción, dificultad para recordar hechos de los días materna (herencia materna) por lo que no 3) Determinación de anticuerpos antiganglicosi-
anteriores, errores en los razonamientos, marcha existe riesgo. dos Anti-GM1 de clase IgG positivos.

273
DESGLOSES MIR
4) Resonancia magnética nuclear craneal normal. de hace 6 meses, consulta por leve dificultad tes, de un mes de evolución, que usted objetiva
5) Potenciales de denervación en el estudio elec- para tragar y diplopía vespertinas desde hace con la exploración física. ¿Cuál de las siguientes
tromiofráfico. unos días. Ingresa en el hospital por sospecha pruebas NO le parecería oportuno solicitar?
de crisis miasténica y se inicia tratamiento de la
Respuesta correcta: 2 misma. Al día siguiente la enfermera avisa a las 1) Electromiografía de fibra muscular.
03 h de la madrugada porque el paciente hace 2) Electromiografía con estimulación repetitiva.
P072 MIR 2006-2007 un ruido extraño al inspirar, como un ronquido 3) Estudio de función autonómica.
suave, el paciente está profundamente dor- 4) Determinación de anticuerpos antirreceptor
mido y muy sudoroso, pero no impresiona de de acetilacolina.
Con respecto a la neuropatía diabética, una de estar fatigado. ¿Qué actitud es más correcta? 5) TC torácico.
estas afirmacioens es FALSA:
1) Tranquilizar a la enfermera y a la familia, pues el Respuesta correcta: 3
1) La mononeuropatía craneal más frecuente es paciente en roncador habitual y está tranquila-
la afectación del VII par. mente dormido. Se debe colocar en decúbito
2) En la afectación del III par craneal se suele con- lateral. P245 MIR 2003-2004
servar la motilidad pupilar. 2) Avisar a cuidados intensivos por sospecha de in-
3) Una afectación frecuente es la presencia de suficiencia respiratoria aguda, para valorar posi- ¿Cuál, entre las siguientes, es la prueba más es-
“síndrome del túnel carpiano”. ble intubación orotraqueal y ventilación asistida. pecífica para el diagnóstico de miastenia grave?
4) El componente sensitivo es el más frecuente 3) Hacer estudio polisomnográfico para descartar
afectado en la plineuropatía diabética. apnea del sueño. 1) Test del cloruro de edrofonio.
5) La miotrofia diabética puede acompañarse de 4) Disminuir la dosis de esteroides: si tiene una 2) Electromiograma con estimulación repetitiva.
un cuadro de caquexia. miopatía esteroidea mejorará. 3) Electromiograma de fibra muscular aislada.
5) Pedir una TC torácica para descartar timoma com- 4) Determinación de anticuerpos antirreceptor
Respuesta correcta: 1 presivo sobre la tráquea asociado a la miastenia. de acetilcolina.
5) TC torácica.
Respuesta correcta: 2
P060 MIR 2004-2005 Respuesta correcta: 4

Un paciente de 35 años refiere pérdida de fuer- P058 MIR 2007-2008


za progresiva en miembros inferiores, de unos 5
días de evolución, dolores musculares y pareste-
sias en pies y manos. En su exploración se aprecia
Una mujer de 34 años refiere diplopía de va-
rias semanas. Unos meses antes había sufrido
T13 Miopatías
únicamente debilidad en los cuatro miembros, diplopía, que remitió espontáneamente. La ex-
de predominio distal y en miembros inferiores, ploración muestra leve ptosis palpebral: P015 MIR 2011-2012
y arreflexia generalizada. Probablemente tiene:
1) El sexo de la paciente hace improbable un Pregunta vinculada a la imagen n.º 8
1) Una miastenia gravis. diagnóstico de miastenia gravis.
2) Una polirradiculoneuritis aguda. 2) La presencia de lesiones desmielinizantes en la Una paciente de 47 años, carnicera, consulta
3) Un proceso expansivo medular cervical. RMN cerebral confirmaría que se trata de mias- por una clínica dos-tres años de evolución de
4) Una mielitis. tenia gravis. debilidad en las manos. Simultáneamente, ex-
5) Una polimiositis. 3) En pacientes de miastenia gravis con afectación perimenta una sensación de “entumecimiento”
ocular limitada, se observa presencia de Ac an- que ha atribuido al contacto con la carne fría
Respuesta correcta: 2 tirreceptores de acetilcolina en sólo el 50%. Este procedente del frigorífico, pero que última-
dato positivo confirmaría el diagnóstico. mente también experimenta en circunstancias
4) Se trata de un síndrome de Horner y la Rx de no relacionadas con su trabajo (abrir botellas,
tórax mostraría un tumor de Pancoast. por ejemplo). El interrogatorio dirigido pone
5) Los inmunosupresores no han demostrado efi- de manifiesto cuatro caídas “casuales” en el
T12 Enfermedades
de la placa motora cacia en la miastenia gravis. último año. La exploración física muestra una
leve ptosis palpebral y debilidad en el giro de
Respuesta correcta: 3 la cabeza, con atrofia de los esternocleidomas-
P065 MIR 2010-2011 toideos. También presenta una discreta paresia
a la flexión dorsal de ambos pies. Los reflejos
Un hombre de 64 años, diagnosticado de mias- P060 MIR 2005-2006 osteotendinosos están presentes y simétricos.
tenia gravis hace 1 año, en tratamiento con Al solicitarle que abra bruscamente las manos
esteroides a dosis bajas (3 mg/día de deflaza- Un hombre de 30 años acude a su consulta por tras mantenerlas fuertemente cerradas duran-
cort) y anticolinesterásicos, asintomático des- presentar debilidad muscular y diplopía fluctuan- te unos segundos, lo hace de una forma lenta

NEUROLOGÍA 274
Y NEUROCIRUGÍA
Neurología y neurocirugía
y dificultosa, tal como puede observarse en la 5) Según la herencia autosómica recesiva, un que solicita una TC (está convencida de que
figura. ¿Cuál de las siguientes entidades es la 25% de sus hijos manifestarán la enfermedad “algo tiene que tener en la cabeza”). ¿Cuál de
que con más probabilidad presenta la paciente? en la infancia, sin distinción de sexos. las siguientes es la causa más plausible de su
cefalea?
1) Distrofia muscular de Duchenne. Respuesta correcta: 3
2) Distrofia miotónica. 1) La migraña.
3) Miotonía congénita. 2) Los trastornos vasculares.
4) Polimiositis. 3) La arteritis de la arteria temporal.
5) Esclerosis lateral amiotrófica.
T14 Cefaleas 4)
5)
La tensional.
La oncológica.
Respuesta correcta: 2
P146 MIR 2013-2014 Respuesta correcta: 4

P016 MIR 2011-2012 Mujer de 40 años que consulta por aproxi-


madamente 20 episodios al día de dolor in- P062 MIR 2010-2011
Pregunta vinculada a la imagen n.º 8 tenso, periocular izquierdo de 15 minutos de
duración, acompañado de intenso lagrimeo y ¿Cuál de las siguientes afirmaciones NO es cier-
¿Cuál de las siguientes pruebas diagnósticas es rinorrea. Su exploración y resonancia magné- ta respecto a la cefalea en racimos?
la más adecuada para establecer el diagnóstico? tica son normales. Su tratamiento de elección
sería: 1) Aparición nocturna.
1) Estudio genético de expansión de tripletes de 2) Presencia de síntomas vegetativos.
ADN. 1) Indometacina. 3) Más frecuente en varones.
2) Imagen por resonancia magnética de los diver- 2) Lamotrigina. 4) Puede volverse crónica.
sos grupos musculares. 3) Verapamilo. 5) Duración media del episodio 8 horas.
3) Potenciales evocados somatosensoriales. 4) Prednisona.
4) Estudio bioquímico completo que incluya cre- 5) Carbonato de litio. Respuesta correcta: 5
tincinasas y aldolasas.
5) Biopsia muscular con estudios inmunohisto- Respuesta correcta: 1
químicos. P067 MIR 2009-2010

Respuesta correcta: 1 P233 MIR 2012-2013 Hombre de 30 años con crisis de dolor intenso
unilateral, diario, de localización fronto-orbi-
¿Qué fármaco NO estaría indicado en el trata- taria derecha, acompañadas de nerviosismo,
P220 MIR 2011-2012 miento preventivo de la migraña? de instauración gradual y remisión en aproxi-
madamente una hora. Dichas crisis suelen
Un hombre presenta una deleción parcial en 1) Sumatriptan. durar 3-4 semanas, reapareciendo a los po-
el gen de la distrofina (cromosoma Xp21) que 2) Topiramato. cos meses. En la exploración durante la crisis
Ie ocasiona la semiología propia de la distro- 3) Propranolol. presenta ptosis, miosis y enoftalmos derecho,
fia muscular de Becker. Acude a la consulta de 4) Flunarizina. edema parpebral y rinorrea derecha. El diag-
genética con su esposa, para valorar los ries- 5) Ácido Valproico. nóstico clínico de sospecha de este paciente
gos de transmisión de la enfermedad. ¿Qué sería:
información correcta será proporcionada en el Respuesta correcta: 1
transcurso del consejo genético? 1) Neuralgia del trigémino.
2) Síndrome de Horner secundario a proceso ex-
1) Según la herencia autosómica dominante, la P080 MIR 2011-2012 pansivo retroocular.
mitad de sus hijos heredarán la enfermedad y 3) Migraña complicada.
sin distinción de sexos. Una mujer de 42 años, casada y con 2 niños en 4) Arteritis de células gigantes.
2) Sus hijas no heredarán la enfermedad, pero to- edad escolar, consulta a su médico de familia 5) Cefalea histamínica (en brotes).
dos sus futuros hijos varones serán portadores por un fuerte dolor de cabeza de unos 7 días
y pueden transmitir la mutación al 50%. de evolución. Aunque ya hace más de un año Respuesta correcta: 5
3) Sus hijos varones no heredarán la enfermedad, que viene teniendo episodios similares, en los
pero todas sus futuras hijas serán portadoras y 2 últimos meses se han agravado notablemen-
pueden transmitir la mutación al 50%. te. El dolor es como un peso que comienza en P055 MIR 2008-2009
4) No hay riesgo: la herencia de la enfermedad es la zona occipital, se extiende a ambas regio-
del tipo mitocondrial, nunca transmitida por nes temporales y apenas se le alivia tomando Una mujer de 26 años acude a Urgencias refi-
los varones. pastillas de 650 mg de paracetamol, por lo riendo una cefalea intensa que ha comenzado

275
DESGLOSES MIR
bruscamente mientras miraba un escaparate P238 MIR 2003-2004 1) Mujeres delgadas de 30-40 años de edad.
hace 2 h y persiste. La exploración neurológi- 2) Hombres obesos de 50-60 años de edad.
ca completa y sistémica son normales. Tiene Un hombre de 30 años presenta episodios de 3) Mujeres obesas de 20-40 años de edad.
antecedentes de migrañas, aunque describe cefalea periocular derecha, que le despiertan 4) Mujeres obesas de 60-70 años de edad.
este dolor como diferente. Señale cuál de las por la noche, muy intensos, de unos 30 mi- 5) Hombres delgados de 30-40 años de edad.
siguientes es correcta: nutos de duración. Le hacen levantarse de la
cama. ¿Cuál sería su sospecha diagnóstica? Respuesta correcta: 3
1) Esta paciente podría tener un problema urgen-
te con más de un 40% de mortalidad. 1) Migraña común.
2) En este caso, una TAC craneal sería una radia- 2) Neuralgia del trigémino. P234 MIR 2008-2009
ción no justificada y una mala gestión de los 3) Cefalea en racimos.
recursos. 4) Sospecharía un tumor cerebral o una hiperten- En el contexto de una lesión en el hemisferio ce-
3) Lo más adecuado sería instaurar tratamiento sión intracraneal. rebral derecho, el paciente presenta dilatación de
sintomático con analgesia y remitirla a consul- 5) Migraña basilar. la pupila derecha y alteración en los movimientos
tas de Neurología. del ojo derecho. Estos síntomas son debidos a:
4) Una punción lumbar no aportaría nada porque Respuesta correcta: 3
la paciente está afebril. 1) Herniación subfalcina o subfalcial (cingular).
5) Sólo si tuviera un electrocardiograma normal 2) Herniación transtentorial (uncinada, temporal
sería adecuado iniciar tratamiento del dolor. mesial).
Síndrome de 3) Herniación amigdalar.
Respuesta correcta: 1 T15 hipertensión
intracraneal
4) Acidosis metabólica.
5) Hidrocefalia.

P063 MIR 2005-2006 P229 MIR 2013-2014 Respuesta correcta: 2

Mujer de 34 años diagnosticada de migraña sin Mujer de 29 años, obesa, sin antecedentes
aura, que consulta por episodios de sus cefa- de interés, que consulta por cefalea pulsátil,
leas habituales en número de 4-5 al mes. ¿Cuál
de estos tratamientos NO estaría indicado?
bilateral, intensa, de un mes de evolución,
acompañada de diplopia horizontal y episo-
T16 Hidrocefalia
dios de amaurosis monocular de segundos de
1) Tomar triptantes durante todos los ataques. duración. La exploración es normal, salvo por P057 MIR 2005-2006
2) Utilizar dosis bajas diarias de ergotamina. la presencia de palpidemia bilateral. ¿Cuál de
3) Administrar como profilaxis propanolol. estas pruebas cree que le va a permitir confir- Hombre de 70 años que consulta por un tras-
4) Tratar todos los ataques agudos con naproxeno. mar plenamente su diagnóstico? torno de la marcha y un deterioro cognitivo
5) Utilizar como profilaxis flunaricina. subagudo. Nos indican que el diagnóstico de
1) Ecografía de troncos supraaórticos. presunción del paciente es hidrocefalia a pre-
Respuesta correcta: 2 2) Resonancia magnética de cráneo. sión normal. En este caso, ¿cuál de los siguientes
3) Electroencefalograma. datos NO esperaría encontrar?
4) Punción lumbar.
P063 MIR 2004-2005 5) Potenciales evocados visuales. 1) Una hidrocefalia comunicante con acueducto
de Silvio permeable en la resonancia cerebral.
Un paciente de 54 años refiere desde hace 10 Respuesta correcta: 4 2) Un trastorno de la marcha tipo apráxico.
días una o dos crisis de dolor de ojo derecho, 3) Un LCR con leve elevación de la presión de aper-
con lacrimeo, gran nerviosismo, que le des- tura y con un aumento de células y proteínas.
pierta por la noche, le obliga a salir de la cama P069 MIR 2012-2013 4) La realización de una punción lumbar evacua-
durándole unas dos horas. ¿Cuál de las siguien- dora (30 ml de LCR) puede mejorar la marcha
tes medidas entiende que es más eficaz para Un cuadro de cefalea, que empeora con los del paciente.
calmar el dolor? esfuerzos, cursa con empeoramientos y me- 5) Ausencia de signos de atrofia cortical cerebral.
jorías a lo largo del día y se asocia con edema
1) Oxígeno intranasal. de papila, es típico del síndrome de hiperten- Respuesta correcta: 3
2) Sumatriptan subcutáneo. sión intracraneal. Los estudios de TC/RM son
3) Ibuprofeno oral. normales, confirmándose que se trata de un
4) Tramadol oral. síndrome de hipertensión intracreaneal be- P064 MIR 2005-2006
5) Metamizol intramuscular. nigna o pseudotumor cerebri. ¿A qué tipo
de personas afecta este proceso con más fre- Un paciente de 73 años sufrió un accidente
Respuesta correcta: 2 cuencia? de tráfico con traumatismo craneal del que se

NEUROLOGÍA 276
Y NEUROCIRUGÍA
Neurología y neurocirugía
recuperó. A los tres meses inicia de forma pro- P066 MIR 2010-2011 5) Iniciar quimioterapia para tratar la enfermedad
gresiva alteración de funciones superiores, in- sistémica aunque sea invisible.
continencia urinaria ocasional y su caminar es Mujer de 51 años que acude a Urgencias por
torpe. Probablemente presenta: disminución brusca de agudeza visual, cefalea Respuesta correcta: 4
intensa, náuseas y vómitos. Hipotensa y afebril.
1) Hematoma intraparenquimatoso cerebral tar- Presenta oftalmoparesia derecha por afecta-
dío. ción de III par craneal. Una TC craneal eviden- P235 MIR 2007-2008
2) Hemorragia subaracnoidea. cia una masa en la región selar hiperdensa con
3) Hidrocefalia arreabasortiva. erosión de las apófisis clinoides anteriores. Un hombre de 30 años presentó un tumor
4) Atrofia cerebral postraumática. ¿Cuál es la actitud a seguir? intracerebral frontal derecho que no capta-
5) Tumor cerebral. ba contraste con los métodos de imagen. Se
1) Sospecharía meningitis química derivada de la le realizó una lobectomía frontal. Se le apli-
Respuesta correcta: 3 rotura de un tumor epidermoide selar e iniciaría có radioterapia asociada a temozolomida. Al
de inmediato tratamiento con corticosteroides. cabo de 8 años se produce una recidiva del
2) Indicaría la realización de una angiografía ce- tumor, que ahora muestra, con los métodos
rebral para descartar un aneurisma, ya que lo de imagen, captación de contraste y áreas de
más probable es que estemos ante un caso de necrosis. El estudio histológico estableció el
T17 Tumores
intracraneales hemorragia subaracnoidea y la masa que se diagnóstico de:
evidencia en la TC sea un aneurisma trombo-
sado paraselar. 1) Gangliocitoma.
P043 MIR 2013-2014 3) Ingreso en UCI y tratamiento del shock que 2) Astrocitoma pilocítico.
padece la paciente, y una vez estabilizada 3) Glioblastoma.
Un joven de 20 años, con clínica de ataxia, practicar resonancia cerebral para cirugía pro- 4) Meduloblastoma.
cefaleas y masa sólido-quística en hemisferio gramada. 5) Cordoma.
cerebeloso derecho, es intervenido quirúr- 4) Determinación de bioquímica y hemograma ur-
gicamente, resecandose una lesión que his- gentes, inicio de terapia con corticosteroides a Respuesta correcta: 3
tológicamente muestra células con procesos dosis elevadas y cirugía transesfenoidal urgente.
citoplasmáticos largos y finos, patrón fascicular 5) Punción lumbar para descartar meningitis bac-
y microquístiso, numerosos vasos y fibras de teriana, previo inicio de antibioterapia empíri- P060 MIR 2006-2007
Rosenthal. El diagnóstico anatomopatológico ca. Una vez estabilizada la paciente estudio de
más probable es: la masa selar. Indique cuál de las siguientes son las lesiones
expansivas más frecuentemente halladas en el
1) Astrocitoma pilocítico. Respuesta correcta: 4 ángulo pontocerebeloso:
2) Xantoastrocitoma pleomórfico.
3) Neurocitoma central. 1) Metástasis, gliomas y ependimomas.
4) Liponeurocitoma, P070 MIR 2009-2010 2) Ependimoma, meningioma y osteoma.
5) Enfermedad por priones. 3) Neurinoma (schwannoma), meningioma y tu-
En un paciente de 48 años con adenocarcino- mor epidermoide.
Respuesta correcta: 1 ma pulmonar estadio IIIa tratado hace 8 meses 4) Granuloma de colesterol y neurinoma (schwan-
se diagnostica una lesión intraparenquimato- noma).
sa cerebral de 3 cm de diámetro que produce 5) Quiste aracnoideo cordoma.
P124 MIR 2011-2012 hemiparesia. En resonancia muestra captación
de contraste y produce edema. Un estudio de Respuesta correcta: 3
Hombre de 60 años que acude a urgencias por extensión no muestra indicios de recidiva del
crisis comicial. Ex-fumador desde hace 3 años, tumor primario. La actitud más adecuada será:
sin otros antecedentes de interés. Una TC evi- P064 MIR 2004-2005
dencia metástasis múltiples. ¿Cuál es el origen 1) Radioterapia holocraneal paliativa puesto que
más probable? el estadio actual del tumor no tiene supervi- Un hombre de 60 años presenta un cuadro de
vencia a largo plazo. hemiataxia cerebelosa de dos semanas de evo-
1) Cáncer de cabeza y cuello. 2) Radiocirugía, para controlar la lesión con míni- lución. La RM craneal muestra una imagen en el
2) Cáncer de pulmón. ma morbilidad. hemisferio cerebeloso derecho que capta con-
3) Cáncer de vejiga urinaria. 3) Biopsia estereotáxica, dado que el diagnóstico traste en anillo y desplaza el cuarto ventrículo.
4) Cáncer de colon. es incierto y las opciones terapéuticas muy di- El diagnóstico más probable es:
5) Cáncer de páncreas. ferentes.
4) Extirpación quirúrgica mediante craneotomía 1) Metástasis cerebral.
Respuesta correcta: 2 abierta seguida de radioterapia. 2) Glioblastoma multiforme.

277
DESGLOSES MIR
3) Linfoma cerebral primario. 5) Trombosis del seno venoso sagital. en la escala de Glasgow de 5 puntos. Se rea-
4) Hemangioblastoma cerebeloso. lizaron diversas TAC cerebrales que fueron
5) Astrocitoma pilocítico. Respuesta correcta: 2 informadas repetidamente como normales.
Un RNM realizado al cabo de una semana del
Respuesta correcta: 1 accidente detectó una zona de contrusión
P230 MIR 2009-2010 hemorrágica a nivel del esplenio del cuerpo
calloso. Al cabo de un mes del traumatismo, la
P246 MIR 2003-2004 Un paciente de 35 años es traído al Servicio situación del paciente persistía inmodificada,
de Urgencia tras sufrir un accidente de tráfico. con una puntuación de 5 puntos de Glasgow,
El tumor más frecuente a nivel del ángulo pon- No abre los ojos ante estímulos dolorosos. No presentando diversos episodios de hiperhi-
tocerebeloso es el: obedece órdenes y emite sonidos incompren- drosis e hiperpirexia, y no detectándose otras
sibles. Su brazo derecho aparece deformado lesiones que la citada en sucesivos controles
1) Glioma de tronco cerebral. y no responde a estímulos dolorosos; sin em- radiológicos:
2) Neurinoma de V par. bargo, su mano izquierda responde en un mo-
3) Colesteatoma del poro acústico. vimiento intencionado hacia el estímulo. Su 1) La causa más frecuente de coma mantenido
4) Meningioma de punta de peñasco. escala de coma de Glasgow (GCS) es: en un traumático craneal es el estatus epilepti-
5) Neurinoma del acústico. co y debería iniciarse tratamiento para ello.
1) 12 2) Creo que la situación clínica del paciente obe-
Respuesta correcta: 5 2) 9 dece a causas no neurológicas.
3) 5 3) Creo que el paciente presenta una lesión axo-
4) 8 nal difusa.
5) 2 4) Es imposible que un paciente en coma presen-
te una TAC normal.
T18 Traumatismos
craneoencefálicos Respuesta correcta: 4 5) Debería procederse a la evacuación quirúrgica
de la lesión del cuerpo calloso.

P149 MIR 2013-2014 P054 MIR 2008-2009 Respuesta correcta: 3

Tras un accidente de tráfico un paciente de En relación con las fracturas postraumáticas de


38 años ingresa en UCI en coma. Tras varios la base craneal, es INCORRECTO que: P094 MIR 2005-2006
días el paciente no mejora neurológicamen-
te y en la TAC se visualizan lesiones punti- 1) Afectan con más frecuencia la fosa anterior y el La evaluación neurológica inicial de un pacien-
formes hemorrágicas en cuerpo calloso y en peñasco del hueso temporal. te con traumatismo craneoencefálico revela
unión cortico-subcortical. ¿Cuál es su diag- 2) La presencia de hematoma periocular o re- que solo abre los ojos ante un estímulo doloro-
nóstico? troauricular debe hacer sospechar su existen- so y emite sonidos incomprensibles, pero loca-
cia. liza el dolor. Señale su puntuación en la Escala
1) Hematoma subdural agudo. 3) La ausencia de salida de líquido cefalorraquí- de Coma de Glasgow:
2) Púrpura trombocitopénica. deo por nariz u oído excluye la existencia de
3) Contusión hemorrágica cerebral. brecha dural acompañante y por tanto, el ries- 1) 6 puntos.
4) Lesión axonal difusa grave. go de meningitis. 2) 4 puntos.
5) Encefalopatía hipóxico-isquémica. 4) El germen más frecuentemente implicado en 3) 3 puntos.
las meningitis secundarias a brecha dural pos- 4) 9 puntos.
Respuesta correcta: 4 traumática es el neumococo. 5) 12 puntos.
5) Un paciente consciente, orientado y sin foca-
lidad neurológica, pero con sospecha de frac- Respuesta correcta: 4
P081 MIR 2011-2012 tura de base debe ser ingresado para observa-
ción durante, al menos, 24 horas.
Si después de un TCE apareciese exoftalmos
pulsátil, soplo audible, intensa quemosis Respuesta correcta: 3
conjuntival, nos indicaría la posible existen- T20 Patología
raquimedular
cia de:
P061 MIR 2006-2007
1) Hematoma subdural crónico. P047 MIR 2013-2014
2) Fístula carótida-cavernosa. Paciente de 25 años, que sufrió un traumatis-
3) Hidrocefalia postraumática. mo craneoencefálico de alta energía, ingresó ¿De qué raíz depende predominantemente la
4) Fractura de base craneal anterior. en el hospital en coma con una valoración inervación sensitiva del primer dedo del pie?

NEUROLOGÍA 278
Y NEUROCIRUGÍA
Neurología y neurocirugía
1) L3. 1) Hipoestesia en la planta del pie derecho. 2) Quinto nervio cervical y quinto torácico.
2) L4. 2) Pérdida del reflejo rotuliano derecho. 3) Cuarto nervio cervical y quinto torácico.
3) L5. 3) Debilidad en el músculo soleo derecho. 4) Quinto nervio cervical y cuarto torácico.
4) S1. 4) Pérdida del reflejo aquíleo derecho. 5) Tercer nervio cervical y cuarto torácico.
5) L2. 5) Pérdida de sensibilidad en la cara dorsal del pie
derecho. Respuesta correcta: 4
Respuesta correcta: 3
Respuesta correcta: 5
P052 MIR 2008-2009
P155 MIR 2013-2014
P079 MIR 2012-2013 ¿Cuál de los siguientes síntomas o signos NO
Un hombre de 70 años presenta desde hace unos son característicos del síndrome de la esteno-
meses, sin traumatismo previo, dolor lumbar, difi- La estenosis espondilótica del canal vertebral sis del canal lumbar?
cultad para la marcha, pérdida de fuerza y pareste- lumbar se caracteriza por:
sias en miembros inferiores, teniendo que pararse 1) Mejoría de los síntomas con la flexión del tronco.
a los pocos metros de iniciar la misma. El paciente 1) Lesiones medulares hiperintensas en el estu- 2) Hiperreflexia rotuliana y aquilea bilateral.
cada vez más va inclinando el tronco hacia adelan- dio de resonancia magnética. 3) Claudicación de la marcha.
te. ¿Cuál es el diagnóstico más probable? 2) Dolor en las pantorrillas que aparece en decú- 4) Pulsos pedios presentes.
bito y mejora al caminar. 5) Parestesias de localización gemelar.
1) Fractura vertebral lumbar osteoporótica. 3) Paraplejia espástica de evolución crónica-pro-
2) Hernia discal central L5-S1. gresiva. Respuesta correcta: 2
3) Espondilodiscitis. 4) Mielopatía transversa aguda con incontinencia
4) Estenosis del canal lumbar. de esfínteres.
5) Escoliosis lumbar degenerativa. 5) Lumbalgia irradiada a nalgas y muslos, induci- P059 MIR 2008-2009
da en bipedestación prolongada, que desapa-
Respuesta correcta: 4 rece en reposo. Mujer de 78 años, hipertensa, diabética y an-
ticoagulada con acenocumarol por fibrilación
Respuesta correcta: 5 auricular. Consulta por un cuadro agudo, sin
P027 MIR 2012-2013 trauma previo, de intenso dolor cérvico-dorsal
irradiado a miembro superior derecho que au-
Pregunta vinculada a la imagen n.° 14 P087 MIR 2010-2011 menta con las maniobras de Valsalva y debilidad
rápidamente progresiva de miembros derechos.
Se trata de un enfermo de 42 años qe presenta ¿Cuál de las siguientes no es una indicación En la exploración destaca hemiparesia derecha
un dolor lumbar intenso, irradiado por glúteo, de tratamiento quirúrgico de una hernia discal y síndrome de Horner también derecho. No hay
cara dorsal de muslo y dorsolateral de la pierna lumbar? afectación facial. ¿Cuál de los siguientes diag-
derecha. En la exploración (imagen n.° 14) en- nósticos le parece más probable?
contramos un pie derecho caído (foto 1) y ocurre 1) Deterioro progresivo sensitivo.
lo que puede ver cuando le pedimos que levante 2) Síndrome de cola de caballo. 1) Hematoma epidural cervical.
las puntas de los pies (foto 2). Debe sospechar: 3) Fracaso de tratamiento conservador más de 2 2) Aneurisma de aorta ascendente.
semanas. 3) Hernia de Chiari de tipo I.
1) Neuropatía del nervio ciático común. 4) Deterioro progresivo motor. 4) Trombosis de la arteria vertebral izquierda.
2) Neuropatía del nervio ciático poplíteo externo 5) Cuadro de déficit motor asociado a trastorno 5) Disección de la arteria carótida interna iz-
o peroneal. del control esfinteriano. quierda.
3) Radículopatía S1 derecha.
4) Radículopatía L5 derecha. Respuesta correcta: 3 Respuesta correcta: 1
5) Hernia discal L3-L4.

Respuesta correcta: 4 P210 MIR 2010-2011 P056 MIR 2006-2007

Una mujer de 42 años con cáncer de mama Un hombre de 80 años refiere desde hace
P028 MIR 2012-2013 presenta metástasis a nivel de los agujeros in- nueve meses dolor intermitente en piernas y
tervertebrales entre la 4.ª y 5.ª vértebras cer- parestesias que aparecen después de caminar
Pregunta vinculada a la imagen n.° 14 vicales y entre la 4.ª y 5.ª vértebras torácicas. 100-200 metros. Los síntomas comienzan en
¿Qué nervios espinales estarán lesionados? las zonas distales de las extermidades inferio-
¿Qué otros hallazgos esperaría encontrar en la res, ascienden a los glúteos y se acompañan de
exploración física del paciente anterior? 1) Cuarto nervio cervical y cuarto torácico. dolor lumbar. Los episodios son más frecuentes

279
DESGLOSES MIR
cuando camina cuesta abajo que cuesta arriba, P093 MIR 2005-2006 1) Se da con más frecuencia en varones.
y se alivia al sentarse o ponerse en cuclillas, o 2) Es frecuente en ancianos.
flexionándose hacia delante mientras sigue Una espondilolistesis grave tipo IV de L5 en 3) A la exploración, los pacientes presentan posi-
caminando. La exploración neurológica es nor- un enfermo de 16 años, ¿qué expresión clínica tividad para las maniobras de Lassègue y Bra-
mal. ¿Cuál de los siguientes diagnósticos es el puede producir, entre las siguientes, con ma- gard.
más probable? yor probabilidad? 4) Sus síntomas consisten en dolor, parestesias y
sensación de debilidad en los miembros infe-
1) Disco torácico herniado. 1) Paraplejia. riores y se desencadenan con la marcha.
2) Estenosis espinal lumbar. 2) Deformidad cifótica lumbo-sacra. 5) Los síntomas mejoran sentándose.
3) Estenosis de la arteria ilíaca. 3) Lordosis torácica.
4) Miastenia gravis. 4) Parálisis cuádriceps. Respuesta correcta: 3
5) Neuropatía periférica desmielinizante. 5) Pérdida fuerza del pie.

Respuesta correcta: 2 Respuesta correcta: 5 P247 MIR 2003-2004

La displasia cráneo-cervical es una malforma-


P078 MIR 2006-2007 P087 MIR 2004-2005 ción heterogénea de la base del cráneo, que
va desde una mera impresión basiliar a una
Un enfermo de 40 años que refleja dolor in- Ante un paciente varón de 41 años, que acu- acusada deformidad que incluye platibasia,
tenso a nivel de la región posterior del muslo de a urgencias con un dolor muy intenso en convexobasia, acortamiento del clivus y apla-
y pierna derecha con reflejo aquileo abolido, región lumbar baja, de 12 horas de evolución, namiento de la fosa posterior. ¿Cuál es el me-
el diagnóstico primero que habrá que pensar y cuya exploración física general es normal. canismo patogénico de esta displasia?
es: ¿Qué actitud terapéutica está contraindicada?
1) Trastorno en el desarrollo del cráneo membra-
1) Hernia discal L5-S1. 1) Reposo absoluto en cama durante 10 días. noso.
2) Fractura patológica L5. 2) Educación postural. 2) Trastorno en el desarrollo del cráneo encon-
3) Metástasis L5. 3) Control del dolor con analgésicos y/o AINE. dral.
4) Discitis L3-L. 4) Ejercicios suaves. 3) Falta de fusión cráneo-vertebral.
5) Tuberculosis. 5) Relajantes musculares. 4) Inmadurez del cerebelo.
5) Descenso excesivo del tronco cerebral.
Respuesta correcta: 1 Respuesta correcta: 1
Respuesta correcta: 2

P062 MIR 2005-2006 P094 MIR 2004-2005

Un paciente de 62 años presenta una historia Un albañil sufre un accidente laboral precipi-
de cervicalgia irradiada a hombros. Desde hace tándose desde 6 metros de altura. Presenta T22 Neurocirugía
funcional
un año presenta dificultad progresiva para ca- un importante dolor a nivel lumbar y déficit
minar añadiéndose dolor en brazo derecho. A de extensión contra gravedad de los dedos
la exploración presenta un reflejo bicipital abo- del pie derecho. Habrá que pensar que puede P224 MIR 2011-2012
lido y unos reflejos osteotendinosos policinéti- tener:
cos en piernas: Conociendo la función de las estructuras re-
1) Una lesión de la raíz L-3. lacionadas con la percepción del dolor y asu-
1) Creo que tiene un tumor medular y le solicita- 2) Una lesión de la raíz L-4. miendo que la estimulación eléctrica puede
ría una RNM cervical. 3) Una lesión de la raíz S-1. bloquear la transmisión del dolor, ¿qué estruc-
2) Creo que tiene una hernia discal con espondi- 4) Una lesión de la raíz L-5. tura no debería ser estimulada eléctricamente
losis y le solicitaría RNM cervical. 5) Una lesión de la raíz S-2. para el tratamiento del dolor?
3) Probablemente tiene una siringomielia y le so-
licitaría una RNM. Respuesta correcta: 4 1) Corteza sensorial primaria.
4) Creo que tiene espondilosis cervical y le solici- 2) Núcleos intralaminares del tálamo.
taría una TAC de columna cervical. 3) Sustancia gris periacueductual/periventricular.
5) Creo que un estudio radiológico simple de P014 MIR 2003-2004 4) Columnas dorsales de la médula espinal.
columna y un tratamiento con AINE es lo ade- 5) Estimulación eléctrica transcutánea de aferen-
cuado. ¿Cuál de las siguientes afirmaciones es FALSA cias primarias de bajo umbral.
con respecto a la estenosis del canal espinal
Respuesta correcta: 2 lumbar? Respuesta correcta: 1

NEUROLOGÍA 280
Y NEUROCIRUGÍA
Neurología y neurocirugía
P059 MIR 2007-2008
Otros temas
En una paciente de 68 años con dolor facial en
el que se sospecha una neuralgia idiopática de P223 MIR 2010-2011
la segunda rama del nervio trigémino derecho,
una de las siguientes respuestas NO sería co- Sólo uno de los siguientes es un derivado de la
rrecta: cresta neural:

1) Hipoestesia en la zona malar derecha. 1) Cápsula suprarrenal.


2) La duración del dolor es de unos pocos segun- 2) Intestino.
dos. 3) Miocardio.
3) Los paroxismos dolorosos pueden desencade- 4) Cerebelo.
narse al tocar suavemente una zona facial. 5) Huesos del cráneo.
4) El reflejo corneal es normal.
5) Indicaría tratamiento con carbamacepina. Respuesta correcta: ANU

Respuesta correcta: 1

P062 MIR 2006-2007

Mujer de 72 años que presenta desde hace 2


meses dolor paroxístico en el labio superior y
mejilla derechos. ¿Cuál de los siguientes datos
iría en CONTRA del diagnóstico de neuralgia
esencial de trigémino?

1) El dolor se irradia a veces a nivel retroauricular


y nucal del mismo lado.
2) El dolor está ausente durante el sueño.
3) Mejoría del dolor con carbamacepina.
4) La masticación desencadena a veces el dolor.
5) La exploración de los pares craneales es nor-
mal.

Respuesta correcta: 1

281
DESGLOSES
MIR
Imagen 1

Imágenes MIR
2009 -2010

Imagen 2

Imagen 3
DESGLOSES MIR

Imagen 4

Imagen 7

Imagen 5

Imagen 6

Imagen 8

IMÁGENES MIR 380


2009 -2010
Imágenes MIR 2009 -2010

Imagen 9

Imagen 11

Imagen 12

Imagen 10

381
DESGLOSES
MIR

Imagen 1

Imágenes MIR
2010 -2011

Imagen 2
DESGLOSES MIR

Imagen 3

Imagen 5

Imagen 4

Imagen 6

Imagen 7

IMÁGENES MIR 384


2010 -2011
Imágenes MIR 2010 -2011

Imagen 10

Imagen 8

Imagen 9

385
DESGLOSES MIR

Imagen 13

Imagen 11

Imagen 14

Imagen 12

Imagen 15

IMÁGENES MIR 386


2009 -2010
DESGLOSES Imagen 1

MIR

Imágenes MIR
2011 -2012
Imagen 2

Imagen 3
DESGLOSES MIR

Imagen 4

Imagen 6

Imagen 5

Imagen 7

IMÁGENES MIR 388


2011 -2012
Imágenes MIR 2011 -2012

Imagen 8

Imagen 11

Imagen 9

Imagen 12

Imagen 10

Imagen 13

389
DESGLOSES MIR

Imagen 14

Imagen 15

IMÁGENES MIR 390


2011 -2012
DESGLOSES
MIR Imagen 1

Imágenes MIR
2012 -2013

Imagen 2

Imagen 3
DESGLOSES MIR

Imagen 4

Imagen 7

Imagen 5

Imagen 8

Imagen 6

IMÁGENES MIR 392


2012 -2013
Imágenes MIR 2012 -2013

Imagen 11

Imagen 9

Imagen 12

Imagen 10

393
DESGLOSES MIR

Imagen 15

Imagen 13

Imagen 16

Imagen 14

Imagen 17

IMÁGENES MIR 394


2012 -2013
DESGLOSES
MIR
Imagen 1

Imágenes MIR
2013 -2014

Imagen 2
DESGLOSES MIR

Imagen 4

Imagen 6

Imagen 3

Imagen 7

Imagen 5

IMÁGENES MIR 396


2013 -2014
Imágenes MIR 2013 -2014

Imagen 8

Imagen 11

Imagen 9

Imagen 12

Imagen 10

397
DESGLOSES MIR

Imagen 13 Imagen 16

Imagen 14

Imagen 15

IMÁGENES MIR 398


2013 -2014
Imágenes MIR 2013 -2014

Imagen 17

399

Você também pode gostar